You are on page 1of 186

HANDBOOK FOR

DERMATOLOGY RESIDENTS
FIRST EDITION

USMAN RAFIQ
ALL RIGHTS RESERVED
No part of this book may be reproduced in any form by
Photostat, microfilm, xerography or by any other means, or
incorporated into any information retrieval system, electronic or
mechanical, without written consent of copyright author.
About the Author
Dr Usman Rafiq is a graduate of Ayub medical college, session
2012. He is a former three times Gold medalist and only student
from his batch to receive Khyber Medical University Gold medal
.He is an author of four titles including 3 on subject of
Dermatology, he is working as consultant dermatologist and in
charge Dermatology unit at King Abdullah Teaching hospital
Mansehra, moreover he is currently the youngest consultant
dermatologist in Health department, KPK.
Dedicated with gratitude to my supervisor

My teacher and my guide

Wing Commander Dr Muneeza Rizwan


Assistant Professor of Dermatology

Fazaia medical college

Head of Dermatology department

PAF hospital Islamabad

And the person who introduced me to this field

Dr Syed Kamran Rizvi


Associate Professor of Dermatology

Dermatology department

Ayub Medical Institute


PREFACE

For the past few months I have been working on this book and
now that it is in your hands, I am a happier person because today
I am passing on, to my juniors, my personal understanding and
experience of preparation for clinical exam. Let it be borne in
mind that it is not all, that a dermatology resident needs to know
to pass the exam. This book will let the residents know their
requirements during the last weeks of preparation after they
have thoroughly gone through the recommended textbooks,
this is my third effort in the field of medical publication and
there may be some shortcomings. While reading this book,
residents are requested to be mindful of the difference between
raw and refined .Now, it’s your turn to mention improvements,
if any, for which I shall be grateful.

Usman Rafiq

ABOUT THIS BOOK


This book does not represent my own discoveries and matter,
multiple different books have been consulted to extract all that
is presented here to help dermatology residents during the
stressful time of exams, notable books include

• Rook 8th edition


• Rook 9th edition
• Handbook of dermatological drug therapy
• Handbook of systemic drug treatment in dermatology
2nd edition
• European handbook of dermatological treatments
• Derm Notes by Benjamin Barankin
• And various authentic web sources and notes
• Treatments given in this book can be modified
according to latest guidelines.
DIFFICULT ROADS OFTEN LEAD TO BEAUTIFUL
DESTINATIONS!!
TABLE OF CONTENTS

Table of Contents

Section A

SHORT CASES.......................................................
Short Case 1 ........................................................
Incontinentia Pigmenti .......................................2
Short Case 2 ........................................................
Neonatal LE .......................................................4
Short Case 3 ........................................................
EBA ....................................................................6
Short Case No. 4 ..................................................
Lichen Planus and ................................................
Lichenoid drug eruptions ....................................8
SHORT CASE 5 .....................................................
Mycetoma ........................................................ 12
Short Case 6 ........................................................
TABLE OF CONTENTS

Lupus Vulgaris / DLE ......................................... 15


Short Case 7 ........................................................
Erythema Nodosum .......................................... 17
Short Case No. 8 ..................................................
Lesions in Sporotrichoid Pattern ....................... 20
Short Case 9 ........................................................
XP .................................................................... 22
Short Case 10.......................................................
Calcinosis Cutis ................................................. 26
Short Case 11.......................................................
ILVEN ............................................................... 28
Short Case 12.......................................................
Tuberous Sclerosis ............................................ 30
Short Case 13.......................................................
Cellulitis ........................................................... 32
Short Case 14.......................................................
Darier Disease ................................................. 34
Short Case 15.......................................................
TABLE OF CONTENTS

Lipoid Proteinosis ............................................. 36


Short Case 16.......................................................
Hemangioma / Portwine Stain .......................... 38
Short Case 17………………………………………………………….

Dermatitis Herpetiformis……………………………………41

Short Case 18.......................................................


LEG ULCER ........................................................ 43
Short Case 19.......................................................
Scarring Alopecia of Scalp ................................. 45
Short Case 20.......................................................
Neurofibromatosis Type 1 ................................. 49
Short Case 21.......................................................
Pachonychia Congenita ..................................... 51
Short Case 22.......................................................
Kindler Syndrome ............................................. 53
Short Case 23.......................................................
Gunther’s disease ............................................. 55
Short Case 24 ......................................................
TABLE OF CONTENTS

HHT .................................................................. 57
Short Case 25.......................................................
Lymphangioma Circumscriptum ........................ 59
Short Case 26 .....................................................
Erythema Multiforme ....................................... 61
Section B

LONG CASES

LONG CASES ........................................................


Long Case 1 .........................................................
Pemphigus Vulgaris .......................................... 64
Long Case 2 .........................................................
Connective Tissue Disorders.............................. 74
Long Case 3 .........................................................
Erythroderma ................................................... 94
Long Case 4 .........................................................
Mycosis Fungoides.......................................... 101
Long Case 5 .........................................................
Chronic Actinic Dermatitis............................... 107
TABLE OF CONTENTS

Long Case 6 .........................................................


EPIDERMOLYSIS BULLOSA ............................... 112
Long Case 7 .........................................................
PSORIASIS ...................................................... 118
Long case 8 ..........................................................
LEPROSY
.......... ………………………………………………………..Error!
Bookmark not defined.

Section C
IMPORTANT DRUGS ..................................... 136
1. Retinoids
2. Dapsone
3. Anti Leishmanial agents
4. Colchicine
5. Topical/ Systemic steroids
6. Azathioprine
7. Cyclosporine
8. Methotrexate
9. Clofazimine
10. Thalidomide
TABLE OF CONTENTS

11. Psoralens
12. Antimalarials
13. Ivermectin
14. Anti TB therapy
15. Mycophenolate Mofetil
16. Cyclophosphamide
17. Immuno biological agents
18. Apremilast

Section D
SPECIAL TESTS AND PROCEDURES ................... 165
1. Skin biopsies
2. Diascopy
3. Dermoscopy
4. KOH exam
5. Tzank smear
6. ELISA
7. Immunoflourescence
8. Western Blot
9. Immunoadsorption
10. Extracorporeal photopheresis
11. Photodynamic therapy
12. Plamapheresis
13. Wood's lamp exam
TABLE OF CONTENTS

14. Ziehl-Neelsen staining


SHORT CASES 1

SECTION A

SHORT CASES
SHORT CASES 2

Short Case 1

INCONTINENTIA PIGMENTI

Exam pattern “examine trunk and proceed”


Trunk
▪ Look for Blaschkoid pattern
▪ Evaluate the stage of disease
▪ Examine dorsal aspects of hands and feet
Scalp
▪ Look for Cicatricial alopecia(specially in vertex)
Nails
▪ Look for nail dystrophy
Oral Cavity
▪ Look for adontia, hypodontia, peg shaped teeth,
palatal defects
NOW MOVE ONTO SYSTEMIC EXAMINATION

Eyes
▪ Check vision ( Finger counting)
▪ Strabismus
CNS
▪ Check tone ,power in legs
At end ask the patient to walk and look for gait
Say thanks

Viva questions

Q . What is the mode of inheritance?


• X-linked dominant
Q. Name two X linked disorders?
SHORT CASES 3

▪ Conradi Hunermann Happle syndrome


▪ CHILD syndrome
Q . Differential diagnoses?
▪ Infantile Pemphigoid
▪ Mastocytosis
▪ EB
▪ HSV
▪ Langerhan cell histiocytosis

Q . What are Blaschko lines?


▪ Imaginary lines, representing the pathways of
epidermal cell migration during fetal development

Q . What are langer’s lines?


▪ Lines of skin tension, correspond to collagen
orientation pattern.
Q . Name disorders following Blaschkoid pattern?
1. IP
2. CHILD syndrome
3. ILVEN
4. Sebaceous naevi
5. Naevus achromicus
6. Segmental vitiligo
7. McCune Albright syndrome
8. Linear LP
9. Lichen striatus
10. Linear Morphea
SHORT CASES 4

Q . What kind of infiltrate do u expect in histopath?


▪ Eosinophilic infiltrate with epidermal bullae
Q. Prognosis of disease?
▪ Pigmentation fades away with time
▪ Depends upon the severity of disease
Q .Name stages of IP?
➢ Stage 1 Macules,papules,pustules, vesicles
➢ Stage 2 Verrucous lesions
➢ Stage 3 Pigmentation
➢ Stage 4 Atrophic, hypopigmented, hairless areas
Q .Name CNS features in IP?
• Microcephaly
• Hydrocephaly
• Spastic Paralysis
• Ataxia
• Seizures
• Mental Retardation
Q. Name the gene involved in IP?
NEMO gene

Short Case 2

NEONATAL LE

Examine face and proceed


Face:- look for macular or annular erythema around eye
'Racoon Sign' or 'Panda Sign'
Examine scalp + neck

Trunk
Now more to trunk and look for
SHORT CASES 5

• Purpura / Petechiae
• Any skin changes

↳ examine regional lymph nodes

Abdomen
• Now palpate abdomen for liver and spleen
(2 finger method for infant)

CVS Auscultate heart (heart block)


Say thanks

VIVA Questions

Q. Name DDs- How to differentiate


1. CMV/ Rubella (they are more purpuric)
2. Bloom syndrome (telangiectasias are
present)
3. Rothmund Thomson (poikiloderma)
Q. Name antibodies involved
• Anti RO/LA, Anti U1RNP
Q. What common disorders do mothers of such patients carry ?
• Mothers have SLE, SCLE or leukocytoclastic vasculitis
in 60% cases.
Q. What other skin findings can you see in Neonatal LE ?
• Annular erythemas
• Reticulate pattern
• Subcutaneous lesions
• Telangiectasias
SHORT CASES 6

• Morpheaform lesions
Q. Prognosis of disease?
1. If there is just skin involvement, it resolves by 1st year.
2. In case of heart blocks, 50% need pacemaker.
3. If mother has SLE, there is 2% chance of NLE
4. 25% of future pregnancies can present ȇ NLE
Q. How to monitor the pregnancies?
a. Serial U/S
b. Fetal Echo
Q. Treatment options?
For mothers, high dose steroids / IV immunoglobulin
at 10th week of gestation.

Short Case 3

EBA

Examine feet, legs and proceed


• Examine hands, feet, elbows (sites prone to
trauma)
• Measure size and note no. of blisters or milia or
any Pigmentary changes.
‘’Note any discharge ‘’
• Elicit Nikolsky’s sign + bulla spread sign

Step 2
• Nail changes
• Examine lymph nodes (for lymphoma)
• Eyes
• Scalp
SHORT CASES 7

In systemic exam remember (ASLIM)


→ Amyloidosis
→ SLE
→ Lymphoma
→ IBD
→ Multiple myeloma

A B C
Palpate liver / Ask permission At end ,check for
spleen and examine lower back
Palpate Kidneys perineum tenderness
( Rule out ( water can (For myeloma )
amyloidosis) perineum)

Say thanks

VIVA Questions

Q. Name differential diagnoses?


• Porphyria cutanea tarda
• Localized BP
• Bullous LP
• Nekam's Disease
• Bullous SLE
Q. What are types (clinical) of EBA?
• Inflammatory variant
• Classic variant
SHORT CASES 8

Q. What kind of infiltrate do you expect to see in


histopathology of classical variant?
'Cell poor infiltrate' or neutrophilic infiltrate
Q. Name disorders with cell poor infiltrate?
• EBA
• EB
• PCT
Q. Name first line options for treatment?
Dapose + Systemic steroids
Q. How can you prevent methemoglobinemia?
Add Vit C or Cimetidine ȇ Dapsone

Short Case No. 4

Lichen Planus and


Lichenoid drug eruptions

Examine hands and proceed


• Start with hands and note
o Flat topped papules
o Measure the size
o Look for koebnerization
o Check if wickham’s striae are present / not

Move to
SHORT CASES 9

Oral cavity: Nails: Scalp: Ears:


➢ Reticular ➢ Yellow nail ➢ Pattern (shuster
➢ Atrophic syndrome of hair sign)
➢ Hypertrop like involvem To
-hic ➢ Pterygium -ent rule out
➢ Ulcerative ➢ Trachonych ➢ Violaceo- DLE / LP
➢ look for ia us overlap
dental papules
amalgam ➢ Pseudop-
elade like
➢ Frontal
fibrosing
alopecia

Now examine axillae / groin+extensors of arms → rule out


‘’Graham-little syndrome’’
o Ask patient to raise hand (look for proximal muscle
weakness)
‘’Excluding Myasthenia Gravis’’
Final Step:
o Examine genitalia
o Palpate liver (Hep C)
Say Thanks

VIVA Questions

Q. What is Graham-little syndrome?


➢ Cicatricial alopecia in scalp
➢ Non-scarring alopecia in axillae / groins
➢ Keratosis pilaris like lesions
SHORT CASES 10

Q .How to differentiate b/w 'idiopathic' LP and lichenoid drug


eruptions?

Lichenoid eruptions are

Clinically Histopathologically

o Over photo exposed o More eosinophils


sites o Parakeratosis
o More itchy o Civatte bodies are
o Lack wickham striae located higher
o Spare nails / oral o Focal interruption of
cavity granular layer
o Leave more o Lymphocyte exocytosis
pigmentation

Q. Name same drugs causing lichenoid drug eruptions?


▪ Pencillins
▪ Antimalarials
▪ INH
▪ G-CSF
▪ Interferons
▪ Proton pump inhibitors
▪ Ribavirin
▪ Gold, mercury
▪ NSAIDS
▪ Thiazide diuretics
Q. Name disorders causing twenty-nail dystrophy?
▪ LP
▪ Alopecia
▪ Pemphigus
Q. Name Associations?
▪ HEP C
▪ Alopecia areata
▪ Vitiligo
SHORT CASES 11

▪ IBD
▪ Primary Biliary Cirrhosis
▪ Lichen Sclerosis
▪ Pemphigus
▪ Diabetes
Q. How to differentiate between LP Pemphigoides and Bullous
LP ?
▪ Clinically: Lesions are limited to
already present LP in case of Bullous LP
while in case of LP Pemphigoides there
is wide spread involvement.
▪ Histopathologically: Subepidermal
bullae in both, but on direct
immunoflourescence there is lgG and C3
deposits over DEJ in care of LP
Pemphigoides.
Q. Prognosis?
▪ 50% heal by 9 months
▪ 90% by 18 months
Q. Name syndromes related to LP?
▪ →Vulvovaginitis – Gingivitis Syndrome
▪ →Graham- Little -Piccardi Syndrome
SHORT CASES 12

SHORT CASE 5

Mycetoma

Examine Foot and Proceed


On inspection look for:
▪ Extent
▪ No. of sinuses
▪ No of nodules
▪ Look for other foot

➢ Note temperature -gradient across lesion


➢ Tenderness

➢ Check for depth of involvement ( move nodules


in two directions)

Press the lesion while looking at face of patient


Ideally take a C/S stick to take sample
Look for color of granules and put them on a glass slide
and (move forward)
▪ Now ask the patient to move foot (over nearest
joint) ACTIVE MOVEMENT
▪ Now do passive movements
▪ Check regional lymph node
▪ Check lymph edema
‘’Now move to systemic exam ‘’
Now Auscultate chest (rule out TB + Botryomycosis)

In last step
Palpate liver / spleen (rule out TB)
Say thanks
SHORT CASES 13

VIVA Questions:

Q. Name Differentials:
▪ Actinomycetoma
▪ Eumycetoma
▪ Scrofuloderma
▪ Botryomycosis
▪ Chronic osteomyelitis
Q. Name causative organisms on basis of grains?
Pale Grains Dark Grains
Fusarium Madurella Mycetomatis
Scedosporium Madurella Grisea
Actinomadura Madurae
Nocardia Asteroides
Streptomyces Somaliensis
Q. What is Botryomycosis?
o It’s a chronic inflammatory response to bacterial
infection
o Organism most commonly involved are staph aureus
and Pseudomonas
o Botryo- (means grapes)
o It can involve lungs as well
Q. How will you investigate the case?
➢ Foot x-ray
➢ Smear
→ Gram staining
→ ZN staining
→ KOH staining
➢ Skin biopsies
▪ For histopathology with PAS stain
▪ PCR
▪ Fungal Culture
▪ Mycobacterial culture
SHORT CASES 14

Q. Name agent used to speed up the process of KOH exam ?


DMSO
Q.Name stains that can be used in histopath (while excluding
fungal causes)?
o Fontana Masson
o PAS stain
o Methenamine silver
Q. Name media for mycobacterial growth?
• Bactic media
• MGIT
• LJ Media
Q. Treatment option for Actinomycetoma and Eumycetoma?
Actinomycetoma Dapsone + Streptomycin
Eumycteoma Griseofulvin, Itraconazole, Voriconazole
Q. Dose and duration of Itraconazole therapy?
• 200mg / day till response
Q. Dose for Griseofulvin ?
Microsized upto 1g/day in adults
Ultramicrosized 750mg/day in (divided doses)
Q. How will you treat a case of Scrofuloderma?
Standard TB Therapy for 6 month
Q. How will you give the dose according to weight?
Remember Rifa 4 tab For every 15kgs 1 tab
Q. Name one major side effect of anti TB regimen?
❖ Rifampicin Hepatitis
❖ INH Neuropathy
❖ Pyrazinamide Gout
❖ Ethambutal Retro bulbar neuritis
Q. Name stains used in gram staining?
• Crystal violet dye – Gram Positive bacteria take this
stain (blue)
• Fuchsine (counter stain) Gram negative bacteria take
this stain (pink )
Q. Histopathological changes in Mycetoma ?
• Marked scarring, ulceration, epithelial hyperplasia.
SHORT CASES 15

• Inflammatory response, surrounding the grains


• PAS-stain detects peptidoglycans, giving purple
magenta color.

Short Case 6

Lupus Vulgaris / DLE


(Lesions over face)
(Examine face and proceed)

Examine lesion, note the type and extent

Measure the lesion + palpate


Do diascopsy→ to rule out apples jelly nodules
(Note temp gradient across lesion)

Now examine scalp examine ears


Look for any hair loss ‘Shuster sign’

• Examine oral cavity


• Nails
• Regional lymph nodes
(Now move onto systemic examination)
▪ Chest (auscultation)
▪ Examine abdomen (liver / spleen)
Always look for lesions on rest of the body

SAY THANKS

Q .Name 5 variants of DLE?


SHORT CASES 16

• Chilblain lupus
• Lupus Profundus
• Rosacea Like
• Hypertrophic
• Verrucous
Q .%age of conversion of DLE into SLE?
▪ Localized DLE 6.5%
▪ Disseminated 22%
▪ Chilblain lupus 15 %
▪ Lupus profundus 35%
Q. Name 2 signs related to DLE ?
▪ Tin Tack sign
▪ Shuster sign
Q.Name disorders where apple jelly nodules can be seen?
▪ Lupus Vulgaris
▪ TVC
▪ Lupoid leishmaniasis
Q. Name variants of lupus vulgaris?
▪ Plaque type
▪ Ulcerative and mutilating
▪ Tumor like
▪ Vegetating
▪ Papules, plaque, nodules,
▪ Mucosal
Q.Possible complications with DLE?
▪ Scarring
▪ Contracture
▪ Tissue destruction
▪ Ectropion
▪ Microstomia
▪ Nasal destruction
▪ BCC/SCC/Sarcoma
Q. Differential Diagnoses ?
▪ DLE
▪ Deep mycosis
SHORT CASES 17

▪ LV
▪ Rosacea
▪ Leprosy
▪ Sarcoidosis
Q.How to investigate a case of Lupus vulgaris?
▪ Biopsy for H/P and culture
▪ PCR
▪ To rule out TB focus
i) CXR
ii) ESR
iii) Quantiferon gold test
Q.Name Tuberculids?
Definitive Possible
Lichen scrofulosorum Granulomatous mastitis
Papulonecrotic tuberculi lupus miliaris disseminatus faciei
Nodular
Erythema induratum of Bazin

Short Case 7

Erythema Nodosum

(Examine legs and proceed)


• Look for location, shape, size, extent and color of
nodules Palpate for
• tenderness
• diascopy
• Temperature gradient across lesion
(Now related examination)
1. Look for anemia (pallor)
2. Lymph edema
SHORT CASES 18

3. Throat exam + mucosae of oral cavity (to exclude sore


throat and herpetic gingivostomatitis)
• Lymph node (lymphoma)
• Liver for lymphoma + TB
• Spleen
Tenderness (abdomen) => IBD
At end → ask examiner to examine perineum
(for IBD)
Say Thanks

VIVA Questions

Q. Name major groups in panniculitis?


▪ Lobular
▪ Septal
▪ Mixed
Q. Name DDS?
▪ Erythema nodosum
▪ Polyarteritis nodosa
▪ Nodular Vasculitis
▪ Cold panniculitis
▪ Erythema induratum
Q. What is erythema nodosum contusiformis?
Yellowish green echymotic appearance of EN.
(During evolution)
Q. What are auto inflammatory disorders?
Disorders involving defect in innate immunity e.g.
→ Sapho syndrome
→ Papa syndrome
→ Behcet's disease
→ Familial Mediterranean fever
Q. Name drugs causing EN?
o Gold Minocycline
SHORT CASES 19

o Pencillins Contraceptive pills


o Iodides Sulphonamides
o Bromides
Q. Treatment ladder for EN?
• NSAIDS
• I/L Steroids
• Wet Dressings
• KI
• Colchicine
• Dapsone
• HCQ
• Steroids
• Methotrexate
• Anti TNF agents.
Q. Enlist biopsy findings of Erythema Nodosum?
• Septal Panniculitis
• Septal fibrosis
• Small vessel vasculitis
• Mixed infiltrate
Q. Name Investigations?
• ESR, C-reactive protein
• ASO Titer (streptococcus)
• Blood/Stool culture (Yersinia, streptococcus)
• Chest x-ray, gallium scan (rule out TB/sarcoidosis)
• Biopsy for histopath
Q . Expected time of resolution?
Resolves in 6 weeks (on average)
SHORT CASES 20

Short Case No. 8

Lesions in Sporotrichoid Pattern

This case will cover leishmaniasis, Fish tank granuloma and


Chromoblastomycosis
Examine arm and proceed
Inspection:
• Look for site, no, color and borders
• Pattern of spread
• Examine other arm and rest of body
On palpation:
• Measure size
• Look for lymphangitis
• Check tenderness
• Diascopy
• Check adherence (move lesion in 2 directions)
• Check regional lymph nodes
• Measure limb girth
(Always measure at same landmarks)
Check pulses if ulcerated lesion
Systemic Examination
• Check for pedal edema (if lesion is over legs)
• Auscultate chest to rule out TB, deep mycosis
• Hepatosplenomegaly

VIVA Questions
Name disorders causing sporotrichoid spread (organisms)

Common Bacterial:
Sporotrichosis Anthrax
SHORT CASES 21

M. Marinum Nocardia
Leishmania brasilienses Pseudomonas
Lupus vulgaris
Fungal: Viral:
Cryptococcosis Cowpox Virus
Histoplasmosis
Blastomycosis

Q. Name investigations for leishmaniasis?


• NNN media for culture
• Histopath
• PCR
• Skin smear for LD bodies
Q. What are LD bodies?
• Intracytoplasmic, non flagellated form
• Ellipsoid bodies (2-4 micro meters)
• Seen in macrophages
Q. Name a peculiar histopath finding in sporotrichosis?
Asteroid bodies
Q. Name agents used to treat leishmaniasis?
• Meglumine antimoniate
• Na-stibogluconate
Q. Dose and amount of elemental antimony in both
preparations?
• Meglumine 85mg/ml
(20mg/kg )
• Na-stibogluconate 100mg/ml
Q. Duration of I/M meglumine antimoniate?
20mg/kg - I/M daily x 21 days
Q. Agents that can be used along with I/L therapy?
• Itraconazole
• Rifampicin
• Probenecid
Q. Name few effects of I/M therapy?
SHORT CASES 22

• Pancreatitis (monitor, ECG, LFTs, CP, amylase


weekly till resolution)
• Hepatitis
• ECG changes
• GIT side effects
Q. Name culture media for deep fungal infections?
Brain heat infusion agar If no growth then half
strength corn meal agar
Q. How to treat sporotrichosis?
• KI 3-6ml
• Terbinafine
250mg (for many weeks) continue six weeks after
resolution
• Itraconazole 200mg
Q. Name some organisms causing
Chromoblastomycosiss?
• Fonsacea
• Phialophora Verrucosa
• Rhinocladiella
• Cladiophialophora

Short Case 9

XP

Examine face and proceed?


On inspection-
• Look for lentigines, macules, atrophic scars or any
tumors.
• Examine eyes-pull up/down the lids to check for
symblepharon
SHORT CASES 23

• Examine scalp, note any erythema

(check cervical lymph nodes)

look for changes


• Over hands
• Oral cavity
• Nails
• General look at skeletal system

Systemic Examination
Hearing
(Perform rinne and weber test )
Whisper-test
(Utter something in ear and ask the pt to repeat)
CNS Check tone (spasticity)
Reflexes(hyporeflexia)
Fine touch (neuropathy)
(ask Patient to walk at end)

(Say Thanks)
SHORT CASES 24

Viva Questions

Q. What are expected eye changes?


• Conjunctivitis
• Corneal ulcer
• Ectropion
• Symblepharon
• Pterygium
Q. What is De Sanctis-Cacchione syndrome?
• Ataxia
• Mental retardation
• Hyporeflexia
• Microcephaly
Q. Name defects in XP and XP variants?
• In XP there is ERCC2 gene mutation (NER defect)
• In XP variant there is defective post replication
repair
• (Autosomal recessive)
Q. Possible complications?
• Malignant melanoma, KA, SCC, BCC
• Loss of vision
• Cosmetic disfigurement
• Depression
• Death
Q. Different Diagnosis/Differentiating Points
Bloom syndrome
• No CNS involvement
• Dolicocephaly
• Malar rash
• Nose beaking
Cockayne Syndrome
• Large, hands feet, no poikiloderma
Rothmund-thompson
SHORT CASES 25

• Poikiloderma (trunk)
• Photosensitivity
• Cataracts
Other DDS include - Kindler syndrome, Werner syndrome,
Hydroa vacciniform
Q. Name investigations?
• Unscheduled DNA synthesis assay
• Cell fusion techniques (to identify complimentary
group)
• Pre-natal diagnosis can be done
Q. How to treat a case of XP?
• Photo protection
• Prophylactic retinoids
• Artificial tears
• 5 FU / Chemical peels
• Modern therapies include
• Topical liposomal T4 endonuclease V
enzyme
• Gene therapy
Q. Name any other disorder with ERCC2 involvement?
PIBIDS (trichothiodystrophy)
SHORT CASES 26

Short Case 10

Calcinosis Cutis

Exam scheme
• Examine nodules
• Note size, site and number
• Pinch, check tenderness

• Move them side by side to check adherence

Examine elbows, knees, joints, buttocks

Pinch skin of fingers (rule out scleroderma)


Check ears-to rule out gouty tophi
+ check any swelling /erythema in feet

Examine scalp + oral cavity + face (look for any rash)

Ask the pt to raise hands above head and stand from


sitting position (rule out Dermatomyositis)
Auscultate Chest
(Say Thanks)

VIVA Questions:
SHORT CASES 27

Q .Causes of calcinosis cutis?


• Metastatic
• Dystrophic
• Traumatic
• Idiopathic
• Iatrogenic
• Tumoral calcinosis
• Osteoma cutis
Q. DDS ?
• Calcinosis Cutis
• Rheumatoid Nodules
• Xanthomas
• Gouty Tophi
Q. Treatment options for calcinosis cutis ?
Medicinal Surgical
• I/L steroids CO2 Laser
• Colchicine Surgical Excision
• Probenecid
• Bisphosphonates
• Warfarin
• Na Thiosulfate
• Diltiazem
• Al-hydroxide (2.4g/day)
Q. Name three skin disorders that present with calcinosis cutis?
• Dermatomyositis
• Systemic Sclerosis
• Inflammatory acne
Q. What is prognostic significance of calcinosis cutis in
children (patients of Dermatomyositis)
• Good prognostic sign
SHORT CASES 28

Short Case 11

ILVEN

Examine the lesion, site, size and extension


Measure the size and palpate to feel the lesion

▪ Examine nails of both hands


▪ Examine axillae / groin (rule out linear Darier
disease)
▪ Examine extensors (rule out psoriasis)
▪ Examine Scalp ( Aplasia Cutis)
Now in order to rule out
Epidermal naevus syndrome, examine
Eyes:
▪ Check vision
▪ Check nystagmus
▪ Make H for extra ocular muscles
▪ Ask the pt to close eyes (ptosis)
▪ Check for strabismus
Skeletal
▪ Limb shortening
▪ Kyphoscoliosis
Ask the pt to walk
CNS
▪ Check tone (hypotonia)
▪ Check power in legs / arms
Oral cavity
for teeth abnormality
At end ask pt to walk
(Say Thanks)
SHORT CASES 29

VIVA Questions:-

Q. Histopath findings?
▪ Hyperkeratosis
▪ Papillomatosis
▪ Elongated rete ridges
▪ ↑ pigment in basal layer
Q. Name linear lesions (any 6)?
▪ ILVEN
▪ Lichen striatus
▪ Linear Darier
▪ Linear psoriasis
▪ Linear porokeratosis
▪ Linear LP
Q. Any other DDs that you would like to keep?
▪ Linear and whorled naevoid hypermelanosis
▪ Incontinentia pigmenti
▪ Hystrix lcthyosis
Q. Name characteristics of ILVEN (clinical) Total 6
▪ Dominant in females
▪ Mostly in left leg
▪ Early onset
▪ Pruritic
▪ Psoriasiform
▪ Refractory to treatment
Q. Name treatment options?
▪ 5 FU
▪ Steroids (topical)
▪ Topical systemic retinoids
▪ Tar
▪ CO2 laser
▪ PDL
▪ Cryosurgery
▪ Excision
Q. Possible complications?
SHORT CASES 30

SSC, BBA, KA, Bowen’s disease

Short Case 12

Tuberous Sclerosis

Command – examine face and proceed


Step 1:
▪ Look for color of papules and distribution
▪ General look at face (facial palsy)
▪ Look for forehead plaque
▪ Oral cavity (dental pits)
Step 2:
Now examine trunk - rest of body
▪ Shagreen patch
▪ Nails for periungual fibromas (Koenen tumors)
▪ Ash leaf macules (do diascopy to exclude naevus
anemicus)
Step 3:
Move to systemic exam
o Palpate kidneys (angiomyolipomas)
o Auscultate heart (conduction defects)
Final Step
Step 4:
In CNS (check for 7th nerve palsy)
(Say Thanks)
VIVA Questions
Q .Differential diagnosis of ash leaf macules?
o Naevus anemicus (capillary malformation) + satellite
lesions
o Naevus depigmentosis (well defined borders)
o Segmental vitiligo (wood's lamp exam)
SHORT CASES 31

o Pityriasis Alba
o Progressive macular hypomelanosis
Q. What is hamartoma?
• Benign, focal malformation that arises from tissue of
its origin
Q. Pathology of a ash lead macule?
• Abnormal melanocytes (↓ Tyrosinase activity)
Q. Treatment options?
• Ketogenic diet (high fat/ protein ratio)
• Sirolimus (mTor Kinsae inhibitor)
• Vigabatrin (for epilepsy) (Follow-up every 6-
months-1year)
• CO2 Laser – (for facial lesions)
o Multidisciplinary approach
Q. Prognosis of disease?
▪ Grave
▪ 75% people die by 25 yrs of age
▪ Main causes are intercurrent infections/epilepsy
Q. Gene involved in TSC
• genes encoding proteins hamartin and tuberin
Q. Causes of cobble stone like lesions in oral cavity?
➢ NF 1
➢ Heck's disease
➢ Cowden disease
➢ Darier disease
➢ Crohn’s disease
➢ Nicotine stomatitis
➢ Pyostomatis vegetans
➢ Lipoid proteinosis
➢ PXE
➢ Acanthosis Nigricans
SHORT CASES 32

Short Case 13

Cellulitis

(Examine legs and proceed)


Inspection- site borders, extension
Look for any blisters, scaling, hair loss

Now check:
▪ Pedal edema
▪ Check temperature gradient across lesion
▪ Open toes to look for source (Tinea pedis)
(always check both front and back of legs)
Measure limb girth (same landmarks over both legs)

▪ examine regional lymph nodes


▪ dorsalis pedis / posterior tibial arteries

(say thanks)
SHORT CASES 33

VIVA Questions:

Q. Main causative organisms?


o Mainly streptococcal disease
o Others like staph, H-influenzae, fungi
can be involved
Q. What is Milian's sign?
o Involvement of ear (subcutaneous inflammation)
in case of erysipelas
Q. Differential diagnoses?
o Well's syndrome
o Erysipelas
o Lymph edema
o DVT
Q. What is a Well's syndrome?
▪ Also known as eosinophilic cellulitis
▪ ON H/P – flame figures
▪ Oral steroids, Dapsone, cyclosporine -Tx
Q. How to investigate?
o Blood CP
o Pus swab for C/S
o Blood culture
o ASO titer
o Doppler U/S if suspecting DVT
Q. Possible complications (cellulitis anywhere on body)?
o Sinusitis, cavernous sinus thrombosis, airway
obstruction
o Tongue lymph edema
o Recurrent lymphangitis
o Lymph edema
o Nephritis
SHORT CASES 34

o Myositis
o Abscess
Q. Name treatment options?
o Flucloxacillin
o Clarithromycin
o Benzyl penicillin
o Cephoparazone + sulbactam

Short Case 14

Darier Disease

Examine trunk and proceed


Examine involved area → palpate (harsh feeling)

Examine all seborrhoeic areas

(face, scalp, nasolabial folds, retroauricular areas, axillae


,groins)
Check oral cavity→ look for cobble stone papules
‘’Ask permission for groins ‘’
Nails:
➢ Longitudinal ridging
➢ Red/white lines
➢ V shaped notching
Hands:
➢ Over palms (punctuate keratoderma)
SHORT CASES 35

➢ Over dorsal aspect (skin colored discrete


papules)
Scalp → greasy hair e crusting
(say thanks)

VIVA Questions?

Q. Mutation?
ATP2A2 mutation encoding SERCA2
Q. Name imp histopath findings?
➢ Grains
➢ Corp ronds
➢ Suprabasal acantholysis
Q. Differential Diagnoses?
• Seborrhoeic eczema
• Grover disease
• Acrokeratosis verruciformis of Hopf
• PRP
• Hailey-Hailey disease
• Pemphigus vegetans
Q. Name DDS for palmer pits?
o Gorlin syndrome
o Darier's
o Acrokeratosis verruciformis of Hopf
o Arsenic poisoning
o Keratodermas
Q. Name variants of disease?
• Vesicobullous
• Naevoid
• Flexural
• Nail disease
Q. Aggravating factors?
SHORT CASES 36

Sunlight, lithium, steroids


Q. Complications?
o Herpes
o Pyogenic infections
o SCC (rare)
Q. Oral Tx of choice?
o Acitretin / Cyclosporine

Short Case 15

Lipoid Proteinosis

Command (examine face and proceed)


Step 1:
o Look for pock like scars
o Look for madarosis, moniliform blepharosis
o Examine scalp
o Oral cavity (cobble stone lesions)
o Eyes
Step 2:
Now ask the patient to protrude tongue
(Palpate the tongue, check for mobility)
Step 3:
Ask the patient to take off shirt
➢ Check axillae, elbows, knees, (waxy yellow
lesions, warty lesions)
➢ Examine scrotum (ask permission )
➢ Waxy yellow thickening
Step 4:
Check Hepatosplenomegaly
Final Step
Check tone in legs
SHORT CASES 37

(say thanks)
*If one question is allowed ask name (hoarseness)

Viva Questions

Q. Mutation? Inheritance pattern?


ECM 1 mutation, autosomal recessive (deposition of
amorphous hyaline material in skin, mucosa and
internal organs)
M:F 1:1
Q. Clinical diagnosis? Clinical
o Hoarse voice
o Thickened frenulum
Q. Lab diagnosis?
→ H/P – PAS positive material in dermis
→ decreased immunohistochemical staining for ECM1
→ CT/MRI Brain- temporal lobe calcification
→ PCR
Q. Differential diagnoses?
o Hydroa vacciniform
o EPP
o Xanthomas
o Lichen myxedematosis
o Hypothyroidism
Q. Prognosis of disease?
➢ Normal life span
➢ Benign
➢ Psychological morbidity
Q. Treatment options?
Medicinal:
➢ Oral DMSO
➢ Retinoids
➢ D-Pencillamine
SHORT CASES 38

Cosmetic:
o Dermabrasion
o CO2 laser
o Chemical peels
o Blepharoplasty
Q. Other causes of hoarse voice

o Shabbir syndrome
o Pemphigus
o Leishmaniasis (mucocutaneous)
o Rhinoscleroma
o Relapsing polychondritis

Short Case 16

Hemangioma / Portwine Stain

(Follow same pattern for both)


Inspection:
o Note size, site and number
o Assess whether its plaque or nodule
Now
o Temperature gradient across lesion
o Palpate the lesion for thrill
o Diascopy (portwine stain is non blanchable)

Measure limb girth


Keep same anatomical landmarks on both sides
In general physical exam:
o check pulse / look for anemia
o check edema
SHORT CASES 39

o regional lymph nodes


o check oral cavity (Sturge weber syndrome)
Final step:
o Palpate spleen and liver to rule out
(hemangiomatosis)
o Auscultate Heart (cardiac failure)
❖ Important Note:
If lesions are over face
o Check for eye abnormalities, check for supra
umbilical raphe
If lesions are in pelvis
o Examine genitalia
o Check for neural tube defects
o Make patient walk
(Say Thanks)

VIVA Questions:

Q. Classify Hemangiomas ?
➢Infantile Hemangioma
➢Congenital Hemangioma
➢Tufted Angioma
➢Hemangiopericytoma
➢Kaposiform hemangioendothelioma
➢Angiokeratoma
Q. Name syndromes related to hemangioma?
o PHACES syndrome
o Pelvis syndrome
Q. What is a Kasabach Merritt phenomenon?
Combination of
➢ Hemangioma
➢ Thrombocytopenia
➢ Microangiopathic hemolytic anemia
➢ Consumptive coagulopathy
SHORT CASES 40

Q. Name associations of port wine stain?


➢ Proteus syndrome
➢ Klippel trenaunay syndrome
➢ Sturge-weber syndrome
Q. Name tx options for Infantile hemangioma?
(wait and watch)
▪ Corticosteroids 1-3mg/kg/day
▪ Vincristine 1.5mg/sq m for more than 10 kgs weight,
for less than 10 1/3rd of adult dose
▪ Interferons @ 1-3 MU/m2/day
▪ Propanolol 2mg /day
▪ Excision
▪ IPL
▪ PDL
▪ Imiquimod
Q. Treatment options for portwine stain ?
o Tuneable dye laser
o ND-YAG laser
o IPL
o KTP laser (Resistant cases)
o Cosmetic camouflage
o Control seizures (if Sturge weber)
o Refer to eye department
Q. Possible complications of Hemangioma?
o Ulceration
o Visual disturbance (amblyopia, astigmatism,
strabismus)
o Airway obstruction
o Auditory obstruction
o Cosmetic disfigurement
Q. How to manage a ulcerated Hemangioma?
o Topical mupirocin / systemic antibiotics
o Non adherent gauze
o Oral steroids / PDL
o Analgesics (paracetamol/codeine)
SHORT CASES 41

Q. Name risk factors for infantile Hemangioma?


o White population
o Females
o Premature births
o Low birth weigh
Q. Regression
o 50% by 5 yrs
o 70% by 7 yrs
o 90 % by 9 yrs

Short Case 17 Dermatitis Herpetiformis


Command – examine extensors and proceed
Examine elbows, knees, shoulders, and trunk
Examine hand (wrists) rule out scabies
▪ Palpate the lesions

▪ Perform Nikolsky’s sign and Asboe Hansen sign

Ask permission to examine natal cleft

o Examine hands, nails, scalp, and oral cavity


Now in GPE
o Examine eyes for anemia (pallor)
o Lymph node groups
o Thyroid examination
o
SHORT CASES 42

Systemic examination:
o Palpate liver and spleen
(say thanks)

VIVA Questions:-

Q. Differential diagnoses ?
o Prurigo
o Chronic eczema
o Papular urticaria
o Scabies
Q. Prognosis of DH?
o Normal life expectancy
o 10% cases remit
o May develop lymphomas in long run
Q. What are Howel-jolly bodies?
o Basophilic spot on eosinophilic erythrocytes
o Seen in cases e decreased splenic function
Q. Name disorders that present e neutrophils at papillary
tips?
o DH
o Bullous SLE
o Linear lgA
Q. Name provocating antigens in diet?
o Wheat → Gliadin
o Barley → Hordein
o Rye → Secalin
Q. Name antibodies found in DH?
o Anti reticulin
o Anti Gliadin
o Anti endomysial
o Anti transglutaminase antibodies
Q. Foods to be avoided?
SHORT CASES 43

o Wheat, barley, rye, product of wheat.


Q. Gluten-free food?
o Corn, nuts, eggs, poultry, rice, vegetables
o Fruits
o Pulses
o Fish
Q. Expected findings in indirect immunoflourescence?
o Negative IIF

Short Case 18

LEG ULCER

Examine leg and proceed


A. Inspection:-
o On inspection look for site, no, shape
o Then see margin, edge, floor
Margins
o Transition zone around ulcer
▪ Healing margin →white / blue / red
▪ Fibrosed → thickened
Edge
o Mode of union between floor and margin
▪ Sloping edge (healing ulcer)
▪ Punched out (trophic)
▪ Undermined (TB)
▪ Everted (malignant)
▪ Raised (Rodent ulcer)
Floor
Note-
▪ Granulation tissue
SHORT CASES 44

▪ Slough
▪ Discharge

B. Palpation: -
▪ Temp gradient across lesion
▪ Background skin
▪ Check adherence (move in 2 directions)
(Now focal exam)
▪ Examine regional lymph nodes
▪ Check pulses
▪ Look for varicosities
▪ Do active / passive movements at joints
Finally (Systemic Exam)
▪ Auscultate heart
▪ Auscultate chest (TB)
▪ Palpate liver/spleen (hemolytic anemia)

VIVA Questions:

Q. What is Ankle Brachial pressure index (ABPI)


▪ Measure systolic B.P in both brachial arteries
▪ Measure systolic B.P in both dorsalis pedis arteries
Divide mean of lower by upper Results:
o 1.6 → stiff arteries
o 0.9 – 0.7 → mild arterial
disease
o 0.7-0.4 → moderate disease
o 0.4 or less → severe disease
Q. How to investigate?
▪ FBC, ESR, C Reactive protein (investigations depend
upon clinical picture)
▪ LFTs with electrolytes
▪ FBS
SHORT CASES 45

▪ HBA1C
▪ Serum Albumin
▪ Urine for Bence jones protein
▪ Sickle cell test
▪ Pus Swab for C/S
▪ ECG
▪ Chest x-ray
▪ Doppler u/s
Q. Name drugs causing Pyoderma gangrenosum?
▪ Colony stimulating factors
▪ Interferons
▪ Isotretinoin
▪ Propylthiouracil
(Causes of ulcers can be asked) DDS

Short Case 19

Scarring Alopecia of Scalp

Examine Scalp and Proceed:-


On Inspection:-
o Note exact site on scalp
o Look for characteristics changes
▪ LP → violaceous papules, erythema,
scaling
▪ DLE → Follicular plugs
▪ Pseudopelade → smooth, slightly depressed patterns
▪ Folliculitis Decalvans → pustules, round
areas of hair loss
→ Tufts of 15-20
hair emerging
SHORT CASES 46

▪ Dissecting Cellulitis → Nodules


with cerebriform pattern
Step 2
Palpation:-
▪ Tenderness
▪ Diascopy (avascular areas are paler)
▪ Temp gradient
▪ Now perform hair pull test
o Hold in between thumb and index finger
(15-20 hair)
pull and count no of hair
(Adjuvant examination steps)
▪ Do examine ears →shuster sign
▪ Brows →mucinosis
▪ Oral cavity →LP lesions
▪ Nails →DLE/LP changes
▪ Groin/axillae (Graham-little syndrome)
At Last
• Regional lymph nodes
• Palpate liver to rule out hepatitis
(Say Thanks)

Viva Questions

Q. Simplified classification of scarring alopecia?


❖ Primary
Lymphocytic
▪ DLE
▪ SLE
▪ Pseudopelade
▪ Hot comb alopecia
▪ Alopecia mucinosa
SHORT CASES 47

Neutrophilic
▪ Dissecting cellutitis
▪ Folliculitis decalvans
Mixed
▪ Acne Keloidalis
▪ Acne Necrotica
❖ Secondary:-
▪ Traumatic: Burns, radiations, accidents
▪ Sclerosing LSA, morphea, SS
▪ Granulomatous Sarcoidosis, Necrobiosis
▪ Infections Favus, Kerion, Shingles,
HIV
▪ Neoplastic SCC, BCC, Sarcomas
▪ Developmental Defects
1) Aplasia cutis
2) Naevi
3) Incontinentia pigmenti
4) Icthyosis (lamellar)
Q. What is Rowell syndrome?
Combination of:
▪ Discoid rash/malar rash
▪ Erythema multiforme like lesions
▪ LA antibodies
▪ Speckled ANA pattern
Q. Name two conditions where tin-tack sign is +ive?
▪ DLE
▪ Pemphigus foliaceus
Q. How do Pseudopelade differ from rest of scarring alopecia?
▪ In histopathology of Pseudopelade, elastic tissue is
intact.
Q. How to perform and interpret hair pull test?
▪ Hold 15-20 hair in between index finger and thumb
and pull gently
▪ Count the no of hair that have come off the scalp
▪ If more than 30% hair come off (that’s abnormal)
SHORT CASES 48

Q. Name all the follicular pattern of lichen planus?


▪ Lichen pilanopilaris
▪ Frontal fibrosing alopecia
▪ Graham-little –Piccardi syndrome
Q. Treatment of choice for Folliculitis Decalvans?
▪ Combination of Doxycycline and Rifampicin
▪ For tufting-tar shampoos are recommended
Q. Treatment of choice for actinic LP?
▪ Acitretin
Q. First line – tx for discoid lupus erythematosus?
▪ Antimalarials
Q. Name antimalarials with dosage?
▪ HCQ 6.5 mg/kg
▪ Chloroquine 2.5 mg/kg
▪ Mepacrine 100mg TDS
Q. Name associations of Lichen Nitidis?
▪ Crohn’s disease
▪ Trisomy 21
▪ Congenital megacolon
Q. Nail changes that can be seen in LP?
▪ Trachonychia
▪ Yellow nail syndrome like changes
▪ Pterygium
▪ 20 nail dystrophy
Q. Hair cycle
▪ Anagen 3-5 yrs
▪ Catagen 3-5 weeks
▪ Telogen 3-5 months
SHORT CASES 49

Short Case 20

Neurofibromatosis Type 1

Examine back and proceed


Step 1 Skin Exam
Exam Pattern
▪ Look for café-au-lait macules
▪ Measure size and count number
▪ Look for neurofibromas:
▪ Perform buttonhole sign
▪ Check if they are sessile / pedunculated
▪ Palpate / feel for bag of worm appearance
(plexiform neurofibroma)

Ask patient to raise hands and ask permission to


examine groins
(Crowe sign – axillary / groin freckling)
▪ Check oral cavity – cobble stoning in oral cavity
▪ Eye Lisch nodules
Step 2 -Exclude Endocrine changes
1) Measure Blood Pressure to exclude
pheochromocytoma
2) Look for adisonian pigmentation
3) Gynacomastia
4) chest genitalia (precocious puberty)
Step 3- Skeletal Overview
▪ Microcephaly
▪ Kyphoscoliosis
▪ Limb shortening
▪ ↓ stature
* Ask the patient to walk
SHORT CASES 50

• If time permits check cranial nerve 2 and 6 and palpate


kidneys for Wilm's Tumor
• 2 for vision
• 6 for false localizing sign

VIVA Questions

Q. DDs of Cafe-au-lait macules?


1. McCune Albright syndrome-
Fibrous dysplasia – cafe au lait macules + precocious
puberty.
2. Tuberous sclerosis complex:
3. Watson syndrome
Features of NF1 +Pulmonary valve abnormality
4. Fanconi's Anemia
Bony dysplasia + leukemia
5. Men I and IIB
6. Noonan Syndrome
7. Bloom's syndrome
8. Carney Complex
Q. Name associated tumors of NF 1?
• Wilm's Tumor
• Rhabdomyosarcomas
• Leukemias
• Retinoblastoma
• Malignant melanoma
Q. How to screen for Pheochromocytoma?
▪ Measure blood pressure
▪ Urinary vanillyl mandelic acid (VMA)
Q. Button hole sign?
By pressing neurofibroma in mid, the lesion
disappears.
SHORT CASES 51

Q. Classification of Neurofibromas? Riccardi's classification?


o Type 1 (Von-recklinghausen’s disease)
o Type 2 (Bilateral acoustic neurofibromatosis)
o Type 3 (mixed)
o Type4 (variant)
o Type 5 (late onset)
Q. Gene involved – inheritance pattern?
o Gene encoding protein neurofibromin
(chromosome 17)
o Autosomal dominant
Q. Follow up of patient?
o Every 6 month – 1 yearly follow up.
o Screening of family members
Q. Criteria- 2+ points are required.
o 6+ Café-au-lait macules 5mm + prepubertally
15mm + post puberty
o Axillary / groin freckling – crowe sign
o 2 or more neurofibromas or 1 plexiform neurofibroma
o Bony lesions (sphenoid dysplasia, pseudoarthrosis,
cortical thinning)
o 2 or more Lisch nodules
o Optic glioma
o First degree relative with same criteria

Short Case 21

Pachonychia Congenita

Examine Palms / soles and proceed


Look for pattern of Keratoderma over hands / feet
(focal Keratoderma)
SHORT CASES 52

▪ Examine Nails→ Subungual hyperkeratosis


Increased curvature of nails
▪ Examine oral cavity:- → look for
leukokeratosis
→ look for teeth
▪ Examine rest of skin → look for cysts
o Examine scalp
o Examine extensors – (follicular keratosis)
Additionally- Check for sweating, dry skin, eczematous patches.
If one question is allowed→ ask Name (hoarse voice)
(say thanks)

VIVA Questions:

Q. Classify Keratodermas (simplified) ?


Congenital
▪ Focal
▪ Transgradient
▪ With extracutaneous features
▪ Striate
▪ Punctate
Acquired
▪ Psoriasis
▪ Eczema
▪ PRP
▪ Crusted Scabies
▪ Secondary syphilis
Q. Types of PC?
Type I Jadossohn – Lewandowsky (K6a –K16 mutations)
Type II Jackson – Lawler (K6b –K17 mutations)
Q. What is Haim-Munk syndrome? Features?
a. Papillon-Lefevre syndrome
b. Arachnodactyly
SHORT CASES 53

c. Acroosteolysis
d. Onychogryphosis
Q. What is Clouston syndrome?
o Hidrotic ectodermal dysplasia, autosomal dominant
o Teeth and sweating is normal
o GJB6 gene encoding connexin 30
Q. Treatment options for PC?
o Change foot wear
o Emollients + keratolytics
o Acitretin (25mg/day)
o Botulinum toxin for hyperhidrosis
o Novel therapy (interfering RNAs to regulate Keratin
expression)
Q. What is Keratoderma climactericum?
o Keratoderma of palms/soles
o Women of menopausal age group
o Painful callosities
o Retinoids are tx of choice
o Tropical estrogen cream
Q. Name 3 diseases caused by Connexin mutation?
o Clouston syndrome (connexin 30)
o KID syndrome (connexin 26)
o Erythrokeratoderma Variabilis and progressiva
(connexin 31)

Short Case 22

Kindler Syndrome

*use same pattern for EB


Examine hands and proceed
SHORT CASES 54

Look for blisters, milia, nail changes over hands


Face
Photosensitivity plus poikiloderma
Oral Cavity
Blisters, erosions, dental abnormalities
Scalp
Hair loss( note exact location in scalp)
Actinic keratosis
Examine sites prone to trauma( Knees , elbows)
Eye examination
Erosions, redness
Imp * Poikiloderma +cutaneous atrophy are important features
( Say Thanks)

Viva questions

Q . What is Kindler syndrome?


Kindler syndrome is a form of EB, where there is mixed pattern
of blistering on multiple levels within/beneath the basement
membrane
Q . Inheritance pattern?
Autosomal recessive disorder ( Kindlin 1 gene)
Q . Prognosis of disease ?
• Blistering reduces with age
• Poikiloderma and photosensitivity persists
Q . Treatment of Kindler syndrome?
• Sunscreens
• Good dental hygiene
• Protein diet
• Remove AK and monitor for malignant change
Q .Name investigations?
▪ Antigen maping
▪ Electron microscopy
SHORT CASES 55

▪ Antibody probes
▪ Mutational analysis
Q .Causes of Poikiloderma?
Congenital Acquired
Rothmund _Thomson LE
Kindler syndrome Dermatomyositis
Poikiloderma of Weary Systemic sclerosis
Xeroderma Pigmentosum T cell Lymphoma
Q .What is Poikiloderma of Civatte?
• Benign poikilodermatous condition involving
• Red brown pigmentation with telangiectasias (sides of
neck)
• More common in fair,sun exposed areas, perfumes are
considered as inciting agents ,hormonal factors are
also blamed for this condition.
• Treatment is sun protection , topical retinoids, IPL and
PDL for telangiectasias

Short Case 23

Gunther’s disease

Examine face and proceed


Face and scalp
• Look for maxillary expansion
• Hypertrichosis( malar region, temples, upper arms)
• Intact blisters or crusted lesions
• Destruction of pinnae and nose
Look for changes around oral cavity ( microstomia)

Ask the patient to show teeth


▪ Staining of teeth ( examine with wood’s lamp)
SHORT CASES 56

▪ Examine mucosae to rule out any erosions or blisters

Hands
▪ Scleroderma like changes in hands
▪ Shortening of digits
▪ Pinch the skin to assess thickness
Trunk
▪ Expose the trunk
▪ Look for any blisters
▪ Hyper pigmentation
▪ Skin thickening
At end
▪ Palpate liver and spleen
Say thanks

Viva questions

Q . Mode of inheritance and enzyme involved in Gunther’s


disease?
Autosomal recessive, Uroporphyrinogen 3 Synthase
What is a Pseudoporphyria?
Condition which resembles cutaneous porphyrias but lab
investigations are within normal range .
Q .How do u clinically differentiate between porphyrias and
pseudo porphyrias ?
Sclerodermoid changes and Hypertrichosis are absent in
pesudoporphyrias.
Q . Name agents causing Pseudoporphyrias ?
Naproxen Retinoids Al-hydroxide( dialysis
fluid)
Doxyxycline Contraceptives
Thiazide diuretics UV light
Q . Enlist treatment options for Gunther’s disease ?
SHORT CASES 57

• Photo protection
• Beta carotenes
• Hyper transfusions
• Hydroxyurea
• Bone marrow transplantation
• Gene therapy
Q . What is the dose of Beta Carotenes?
o For adults 180mg per day (Carovet)
o For Children 90mg per day
Q . What is SPF( Sun protection factor)
o MED of UVB light with sunscreen applied/MED of
UVB without sunscreen
o If a normal person develops erythema in 5 mins
without sunscreen with SPF15 sunscreen it would
take (15 into 5=75 mins )
Q . Name agents in sunscreens those protect against UVA?
▪ Avobenzones, Oxybenzones, Zinc oxide ,Titanium
dioxide

Short Case 24

HHT

Examine oral cavity and proceed


▪ Open oral cavity, look for telangiectasias
▪ Retract lips and check mucosae
➢ Examine nasal cavities
➢ Check ears and face
➢ Examine hands/rest of the body
Do diascopy (Non-blanchable lesions)
General physical examination Eyes (pallor)
Systemic Examination
▪ Auscultate Chest
SHORT CASES 58

▪ Auscultate cardiac areas( Cardiac failure)


3rd heart sound might be heard
Palpate Abdomen tenderness
Check for ascites
Say thanks

Viva Questions

Q . What is inheritance pattern in HHT?


▪ Autosomal dominant
Q . Name few neurocutaneous disorders?
▪ Sturge Weber syndrome
▪ Tuberous Sclerosis
▪ NF type 1 and 2
▪ Ataxia –Telangiectasia
▪ HHT
▪ Von Hippel-Lindau disease
Q . Name criteria (features) of HHT?
1. Epistaxis (3 are required)
2. Telangiectasias
3. Visceral lesions
4. Family history
Q .Name cause of Telangiectasias?
Congenital Acquired
▪ Portwine Rosacea
▪ Maffuci syndrome Trauma
▪ HHT Topical steroids
▪ Ataxia –Telangiectasia Carcinoid syndrome
▪ Sturge Weber syndrome Bloom syndrome
▪ Klippel Trenaunay syndrome Systemic Sclerosis
Q .Treatment options for telangiectasias?
▪ IPL
▪ PDL
Q . What is Naevus Flammeus ?
SHORT CASES 59

▪ Other name for port wine stain


▪ Port wine (Portugese wine like )

Short Case 25

Lymphangioma Circumscriptum

Examine and proceed


On Inspection
▪ Look for cluster of firm blisters filled with lymph fluid
( Frog Spawn like)
▪ Measure size and look for extent
▪ Note any oozing ,bleeding or background erythema

Palpation
▪ Feel the temperature gradient (rule out any infection)
▪ Now examine other body sites prone to such lesions
(Shoulders, neck, axillae, groins)
Ask the patient to open mouth
▪ Look for any involvement of tongue
Rule out any cystic hygroma (more common in neck, axillae,
groins)
If cystic Hygroma is suspected, perform transillumination test
with torch
Say thanks

Viva Questions
SHORT CASES 60

Q . Classify Lymphatic malformations?


1. Cystic hygroma
2. Cavernous lymphangioma
3. Lymphangioma Circumscriptum
Q . Investigation of choice to assess the extent?
▪ MRI
Q .Treatment options for Lymphangioma Circumscriptum?
▪ Antibiotics for infected causes
▪ Sclerotherapy
▪ Electrocautery
▪ Electro fulguration
▪ CO2 laser
▪ PDL
▪ Surgical excision (definitive treatment)
Q . Possible complications?
▪ Oozing
▪ Bleeding
▪ Cellulitis
▪ Recurrence
▪ Lymphangiosarcoma
Q . Name differential diagnoses?
▪ Herpes simplex
▪ VEN
▪ Lymphangiectasia
▪ Viral warts
▪ Syringocystadenoma Papilliferum
Q . What is Stewart Treves syndrome?
▪ Lymphangiosarcoma , forming as a result of long
standing lymph edema or as complication of
mastectomy or radiotherapy for breast cancer.
SHORT CASES 61

Short Case 26

Erythema Multiforme

Examine hands and proceed


On Inspection
▪ Look for typical targetoid lesions , atypical papules or
bullous lesions
( acral distribution)
▪ Examine hands , feet and face
Ask the patient to take off his shirt
▪ Examine trunk and look for lymphadenopathy
▪ Now Examine oral cavity, eyes and take permission to
examine genitalia
▪ Do rule out HSV infection

Relevant examination
▪ Examine throat( congestion)
▪ Examine Legs for ( Polyarteritis nodosa)
In last step
Auscultate heart ( rule out Mycoplasma Pneumonia )
Say thanks

Viva Questions
Q . Associations (Infective causes)
▪ Herpes simplex Hepatitis
▪ Mycoplasma HIV
▪ Parapox virus CMV infection
▪ Adenovirus
Q .How to differentiate EM major from SJS?
EM major has
▪ Less than 10 % BSA
▪ Minimal mucosal involvement
SHORT CASES 62

▪ Predominantly acral distribution( typical targets)


▪ Raised targets( bullous EM)
▪ Atypical popular lesions
Q . TEN, SJS ,EM spectrum?
▪ EM major ( less than 10 % BSA)
▪ SJS ( more than 10 % BSA)
▪ SJS-TEN overlap ( 10-30% BSA)
▪ TEN with spots( more than 30% BSA with purpuric
macules and atypical targets)
▪ TEN without spots ( Large epidermal sheets without
purpuric macules)
Q . What is SCORTEN? Prognostic significance
Total seven points
FEATURE SCORE
0 1

Age ≺ 40 ≻40
Malignancy No YES
Heart rate ≺120 ≻120
Serum BUN (mg/dl) ≺28 ≻28
BSA ≺ 10% ≻10%
Serum Bicarbonate (meq/l) ≻ 20 ≺20
Serum Glucose ( mg /dl) ≺ 252 ≻252

▪ Mortality rate is 3 % at score 1


▪ Mortality rate is 90% at score 5 or more
Q . Pronosis of EM?
▪ EM minor heals spontaneously in 2-3 weeks without
scarring
▪ EM major takes up to 6 weeks
▪ Eye complications can occur in EM major
LONG CASES 63

SECTION B

LONG CASES
‫‪LONG CASES‬‬ ‫‪64‬‬

‫‪Long Case 1‬‬ ‫‪Pemphigus Vulgaris‬‬

‫‪History Question‬‬
‫‪ 1‬۔ چھالے کب سے ہیں ؟ کہاں کہاں پر ہیں‪ ،‬پھیلے کس طرح اور کتنے عرصے میں‬
‫پھیلے۔‬
‫‪ 2‬۔ چھا ال نکلنے سے پہلے سرخی تو نہیں ہوتی‪ ،‬خارش تو نہیں ہوتی؟‬
‫‪ 3‬۔ چھاال سخت ہوتا ہے یا نرم‪ ،‬خود بیٹھ جاتا ہے یا اآپ پٹھا دیتے ہیں؟ اور اگر خود‬
‫بیٹھ جاتا ہے تو کتنے عرصے میں بیٹھ جاتا ہے۔‬
‫‪4‬۔ کسی دوا کھانے کے بعد تو شروع نہیں ہوتے؟‬
‫‪ 5‬۔۔ جب زخم ہوتا ہے تو اس کا سائز چھالے سے بڑا ہوتا ہے؟‬
‫‪ 6‬۔ دھوپ میں جانے سے کیا فرق پڑتا ہے۔ ٹھوکر لگنےسے چھاال بنتا ہے۔‬
‫‪ 7‬۔ چھالے کسی خاص شکل میں نکلتے ہیں جیسے گھچے کی شکل میں؟‬
‫‪ 8‬۔ چھالے سے پانی یا خون نکلتا ہے۔ درد‪ ،‬خارش‪،‬جلن)‬
‫‪9‬۔ ٹھیک ہونے کے بعد سفید دانے‪ ،‬رنگ کی تبدیلی تو نہیں چھوڑتے۔‬
‫‪10‬۔ چھالے کبھی منہ‪ ،‬آنکھوں یا بغل میں نکلتے ہیں؟‬
‫‪11‬۔ پیشاب میں تکلیف‪ ،‬آوازمیں تبدیلی‪ ،‬ناخن خراب ہونا‪ ،‬بالوں کا جھڑنا۔‬
‫‪12‬۔ ابھی تک کیا عالج کیا‪ ،‬گولیاں کھانے سے کوئی تکلیف ہوتی ہے جیسے جوڑوں‬
‫میں درد۔‬
‫‪13‬۔ سفید رنگ کی چھوٹی گولیاں کھائی ہیں۔ کوئی ایسی دوا جوہفتے میں ایک بار‬
‫لینی ہو۔‬

‫‪Proceed to Systemic Inquiry‬‬


‫‪Examination:-‬‬
‫‪Examine Scalp, face, oral cavity, nose, rest of the Body‬‬
‫•‬ ‫‪Measure the lesions‬‬
‫•‬ ‫‪Elicit Nikolsky’s sign and and Asboe-Hensen sign‬‬
‫‪In systemic examination, focus more on GIT,‬‬
‫‪musculoskeletal system and do rule our‬‬
‫‪lymphadenopathy and steroid myopathy.‬‬
‫‪Diagnosis‬‬
‫‪A.‬‬ ‫‪Diagnostic Investigations:-‬‬
‫‪o Tzank smear-Acantholytic cells‬‬
LONG CASES 65

o Skin biopsy for histopath and


immunoflourescence
o Indirect immunoflourescence
o ELISA
o Electron microscopy
B. Therapeutic Investigations:-
o FBC e differentials
o RFTs
o LFTs e electrolytes
o ANA to rule out SLE
o TPMT levels
o Blood Pressure
o Ideally DEXA Scan
Treatment:-
o Saline gargles 2-3/day
o KMnO4 bath (1:1000conc) * add 400mg tab
to 4L water to achieve this concentration
o ↑ Fluid intake
o Exercise + no smoking
o Oral triamcinolone gel
o Nystatin or miconazole gel (prophylactic)

Systemic Therapy
First Line Option:-
A. Initiation Phase:-
1-2mg Prednisolone + 1.5mg/kg/day
Azathioprine

Continue for 4-6 weeks (Continue till control of disease)


B. Control of Disease:- When active lesion disappear
C. Consolidation Phase:- When no lesions appear for
2 weeks
D. Maintenance and Tapering
LONG CASES 66

Decrease the dose by 10mg / week till you reach


40mg/day while maintaining Azathioprine dose
same

Once on 40mg/day maintain the dose for one
week

Decrease the dose by 5mg/week till it reaches
20mg/day

Keep the dose at 20mg/day for one week

Decrease the dose by 2.5mg/week till it reaches
5mg/day and maintain it for a week

Decrease dose by 1mg/week and stop the
Prednisolone
Keep Azathioprine at 1.5mg/kg/day and if
remission is maintained for 2 years, It can be
stopped.
Other Agents:-
o Cyclophosphamide (1-3mg/kg/day)
o MM (2g/day)
o Methotrexate (2.5mg weekly)
o Oral Chlorambucil (4-6mg/day)
o Dapsone (50-200mg/day)
Pulsed Therapy:-
Indications are:-
• Severe disease
• Unresponsive disease
• High level of antibody titer
• Requirement of steroids >100mg/day
Method:-
LONG CASES 67

o First Day:- 100mg dexa or 1 g Methyl


Prednisolone in 500ml dextrose (1-2 hours
infusion)
o 2nd Day:- Add 500g Cyclophosphamide
o 3rd Day:- Same as day 1
o Continue 50mg Cyclophosphamide in
between the pulses. After completion of
pulses continue 50mg Cyclophosphamide as
maintenance for 1 year.
➢ Monitoring Pulse Therapy:-
Intra Pulse:- Pulse, BP after 15mins initially then
after every 30mins
After Pulse:- Urea, BSR, electrolytes
Inter Pulse:- Blood CP, LFTs, RFTs, BSR, Urine
R/E, electrolytes and X-ray chest
Side Effects:- Nausea, flushing, mood change,
sleep disturbance, seizures, arrhythmias,
osteonecrosis, sudden death.
Other Options for treatment
IV-Immunoglobulin:-
▪ 2/kg over 4-5 hours infusion
▪ Given over consecutive 3-5days
▪ For several months
Side Effects:-
▪ Chills, dyspnea, tachycardia, HTN,
Nausea,
▪ Muscle + abdominal pain, pyrexia
(Add diphenhydramine +
paracetamol to avoid them)
▪ Bleeding tendency, deranged LFTs,
↓Blood Counts
Rituximab:-
o IgG1-Chimeric monoclonal antibody against
CD20
LONG CASES 68

o target CD20 involved in proliferation of B-


Cells. Mechanism involved is complement
activation and antibody dependent
cytotoxicity.
o IV Dose- 375mg/m2 slow infusion weekly x 4
weeks
o Add 500mg Paracetamol and
Chlorpheniramine 10mg prior to treatment
Side Effects:- Fever, chills, DVT,
hypogammglobulinaemia, neutropenia leading to
infections.
2nd Line Treatment Options:-
o Gold Salts
o Tetracycline 2g/day + 1.5g Nicotinamide
(cholinergic agent)
o Plasmapheresis:- Over (3-4days to remove
harmful antibodies)
o Immunoadsorption:-
▪ 4 procedures are done
consecutively over 4 weeks
▪ SH membranes trap harmful
antibodies
▪ In life threatening cases.
o ECP 2days cycle every month
Q. How you define relapse of disease?
Appearance of 03+ lesions that do not heal
spontaneously within one week, or extension of
already present lesions.

Viva Questions

Q. Name diagnostic tests for Pemphigus?


o Tzank smear
LONG CASES 69

o Skin biopsy for H/p and IF


o Indirect immunoflourescence
o ELISA
o Electron Microscopy
Q. What are therapeutic investigations (done before
starting therapy)?
o FBC e differential
o RFTs / LFTs
o ANA to rule out SLE
o TMPT levels (Azathioprine)
o X-ray chest, urine analysis
o Ideally DEXA Scan
Q. How to do KMnO4 bath?
o Make (1:1000 conc)
o Adding 400mg tab to 4 liters make desired
concentration.
o Take 10-15mins bath daily.
Q. How do you define ‘Control of disease’?
When the active lesions disappear
Q. What is consolidation phase?
o When no new lesion appear for 02weeks, it’s
known as consolidation.
Q. Can we completely stop Prednisolone in patient of
Pemphigus vulgaris?
o If remission is maintained for 02years, by keeping
Azathioprine on, Prednisolone can be
discontinued.
Q. If an active blister is not available, how would you do
IIF?
o Either induce a blister
o Or take urine as sample
Q. What is immunoadsorption?
o In cases where there is life threatening disease or
antibody levels are high, sulphhydryl membranes
(SH) are used to trap harmful antibodies.
LONG CASES 70

Q. How do you define ‘relapse’ of disease?


o Appearance of 3+ lesions that do not heal
spontaneously within one week or extension of
already present lesions.
Q. Name theories regarding pathogenesis of Pemphigus
Vulgaris?
o Steric Hindrance
o Cell-Signaling theory
o Desmoglein-non assembly depletion theory
o Desmoglein-compensation theory
Q. Name drugs causing Pemphigus?
o Captopril
o Pencillin
o Pencillamine
o Cephalosporin
o Contraceptives
o Antimalarials
o Pyrazolone derivatives
Q. What is a desmoglein?
o Desmoglein is a protein, belonging to Cadherin
family
o Adhesion molecule
o Desmoglein 1, 2, 3.
Q. Name adhesion components of desmosomes?
o Desmogleins
o Desmocollins
Q. Name components of adhesion complex?
o Keratin filaments
o Hemidesmosomes
o Anchoring filaments
o Lamina densa
o Anchoring fibrils
Q. Dose of Rituximab?
o 375mg/m2 weekly x 4-8 weeks
LONG CASES 71

Q. How do you define the severity of PV on the basis of


number of lesions?
o Mild (20 lesions)
o Moderate (20-50 lesions)
o Severe (50+ lesions)
Q. Dose of steroid in Bullous Pemphigoid?
o Mild (0.3mg/kg)
o Moderate (0.5mg/kg)
o Severe (0.7-1mg/kg)
Q. Name drugs causing inhibition of Purine synthesis?
o Azathioprine
o Mycophenolate mofetil
Q. Name variants of Bullous Pemphigoid?
o Pemphigoid Nodularis
o Pemphigoid Vegatans
o Localized Pemphigoid
o Localized Vulvar Pemphigoid
o Lichen Planus Pemphigoides
o Induced BP
Q. Why Pemphigus foliaceus is uncommon in neonates?
o In a neonate, Dsg1 and Dsg3 both are co-
expressed in the superficial epidermal layer so
Dsg 3 compensates for Dsg 1 .
Q. Name different signs that are positive in Bullous
disorders?
o Nikolsky’s sign
o Asboe Hensen Sign
o False Nikolsky’s sign (Shekalov sign +ive in sub-
epidermal disorders)
o Pseudo Nikolsky’s sign(Burns, TEN, SJS)
o LUTZ Sign (Bulla spread sign)
Q. Name stain used in T-zank preparation, fixative and
what is an Acantholytic cell?
o Giemsa stain (eosin, glycerin, methanol), fixative
used is Zenker solution
LONG CASES 72

o Acantholytic cells:- Large, round, hypertrophic


nuclei, basophilic cytoplasm.
Q. Prognosis in drug induced Pemphigus?
o If Thiol group – 50% recover spontaneously
o Non-thiol-group 15% recover spontaneously
Q. Pattern of DIF staining in P200-pemphigoid?
o Dermal side (pattern)
Q. Diseases associated with Bullous Pemphigoid?
o Diabetes
o RA
o Parkinsonism
o IBD
o Solid Tumors
o Psoriasis
o Multiple Sclerosis
Q. Bad prognosis sign in BP?
o Old age, high steroid dose, low albumin,
generalized disease
o Low Karnofsky score
Q. What is the best time for taking Deltacortil?
o Best time is 7-8am in the morning
Q. Why Cyclophosphamide is not preferable in young
patients?
o It causes menstrual irregularity in women and
secondary infertility in men.
Q. Indications of Pulse Therapy?
o Severe disease
o Unresponsive disease
o If daily steroid requirement is more than 100mg.
Q. How do you differentiate Paraneoplastic pemphigus
from PV?
In PNP
o More necrotic cells
o Polymorphic lesions
o DIF showing DEJ involvement
LONG CASES 73

o More mucosal involvement


o Association of malignancy
Q. Side Effects of Pulse Therapy?
o Nausea, flushing, mood changes, sleep
disturbance
o Seizures, arrhythmias, osteonecrosis, sudden
death
Q. Mechanism of action of Rituximab?
o IgG1- chimeric monoclonal antibody against
CD20
o CD20 is involved in proliferation of B-cells
o Destroy B-cells by complement activation and
antibody dependent cytotoxicity
Q. Single dose vs split steroid regime?
o For long term Immunosuppression-single
morning dose is preferable
o For short courses split dose regimen in better
Q. Why is Nicotinamide effective in Pemphigus Vulgaris?
o It’s a cholinergic agent, protects against
anticholinergic antibodies
‫‪LONG CASES‬‬ ‫‪74‬‬

‫‪Long Case 2‬‬

‫‪Connective Tissue Disorders‬‬


‫‪(With little modifications, same questions should be asked from‬‬
‫‪a suspected case of C.T disorders‬‬

‫کب سے ہے (کیسے پھیال ‪ ،‬کسی دوا کے بعد شروع تو نہیں ہوا ‪ ،‬بخاریاوزن کی کمی‬
‫‪ ،‬بخار کی کیا شدت رہی ہے۔ کبھی تھرمامیٹر سے بخار چیک کیا۔ سر کے بال تو نہیں‬
‫جھڑے ‪ ،‬سورج چبھتا ھے ۔ سرخ نشان کہاں کہاں موجود ہیں ۔ سورج میں جانے‬
‫سے نشانوں پر کیا فرق پڑتا ہے۔‬
‫۔ ہاتھوں پر سرخی یا چہرے پر سوجھن تو نہیں محسوس ہوئی ۔ ہاتھوں کی انگلیاں‬
‫موٹی تو نہیں ہوئیں‪ ،‬منہ کھولنے میں کوئی دشواری جسم پر کوئی نشان ‪ ،‬کہاں تک‬
‫پھیلے ہوئے۔ ان نشانوں پر کو چھلکا تو نہیں بنتا ۔‬
‫کبھی سرخ نشانوں پر چھالے بنے ۔‬
‫نشان ٹھیک ہو کر رنگت میں کوئی تبدیلی تو نہیں چھوڑتے۔‬
‫ہاتھ نیلے ہونا‪( ،‬ذہنی تنائو یا سردی میں )‬
‫موسم کا ان زخموں پر کیااثر ھوتا ھے‬
‫ٹانگ پر پھوڑے تو نہیں بنے جن کو پھٹانے سےسفید رنگ کا مادہ نکال ہو ۔‬
‫غشی کے دورے ‪ ،‬ناخن میں رنگت کی تبدیلی یا ناخنوں کا موٹا ہونا۔‬
‫‪,‬‬
‫‪To rule out MCTDs and other CTDs individually ASK‬‬
‫بالوں کا جھڑنا ‪،‬آنکھوں میں خشکی یا ریت محسوس ہونا۔‬
‫منہ خشک ہونا‪ ،‬منہ میں چھالے بننا‪،‬‬
‫آنکھوں کے گرد سرخی آئی ھو یا ‪,‬چہرے پر سرخی کو دھوپ میں کیا فرق پڑتا ہے‬
‫ھاتھوں پر آئی ھو۔‬
‫سانس لینے میں تکلیف ‪ ،‬چلنے سے سانس پھولنا ‪ ،‬کھانسی رہنا ‪،‬خشک یا بلغم آتا‬
‫ھے۔‬
‫‪ 6‬۔ اٹھنے میں مشکل کا سامنا‪ ،‬کروٹ بدلنے میں تکلیف کا ھونا‬
‫سردی گرمی لگنا ‪ ،‬گلے میں کوئی گلٹی محسوس ہونا‪،‬‬
‫پیروں میں گرمی سے درد ہونا‪ ،‬ٹانگ پر سرخ خون کے دھبوں جیسے نشان ھونا‬
LONG CASES 75

،‫ پشاب کا رنگ کاال ہونا‬،‫پیٹ خراب رہنایا قبض رہنا‬


،‫ جھاگ یا بدبو کا ہونا‬، ‫چھوٹے پشاب میں جلن‬
، ‫ جوڑوں پر سرخی اور اکڑن‬، ‫ اگر ہے تو کونسے‬، ‫جوڑوں میں درد‬
‫اکڑن صبح اٹھ کر کتنی دیر میں چلی جاتی ہے۔‬
، ‫ سفید چاول جیسی گولیاں کھائی ہیں‬، ‫کوئی دوا لی ہے اس مرض کے لئے‬
‫گولیانلینے کے بعد کوئی کمزوری یا تکلیف ۔‬
‫اس بیماری کی وجہ سے ہسپتال میں داخلہ ہوا ہے۔ اگر ہوا تو کتنی دیر تک اور عالج‬
‫سے کیا فرق پڑتا۔‬

Proceed to rest of the History accordingly


2A Systemic Lupus Erythematosus
Management
Investigations:-
▪ Blood CP – anemia (hemolytic anemia,
anemia of chronic disease, blood loss,
iron deficiency, renal failure)
▪ Leucopenia –Neutropenia, Lymphopenia
▪ Thrombocytopenia
o ESR ↑ed
o CRP +ive (If current infection)
o Complement levels – (C3, C4) ↓ed (Genetic or due
to increased consumption)
o RFTs
o LFTs
o Creatine Kinase levels (myositis + overlap
syndrome)
o Urinalysis – Proteinuria, hematuria, casts
Auto antibodies:-
o ANA (95%) significant titer (1:160)
o Anti Ds DNA (60%)-nephritis more with this
antibody
o Anti smith antibodies (25%) – most specific,
associated e nephritis, CNS and vasculitis
o Anti RO (30%) Sjogren syndrome, renal disease,
SCLE, neonatal LE)
LONG CASES 76

o Anti Histone (Drug induced lupus)


o Anti Ribosomal P (25%) CNS features
o U1RNP (MCTDs)
o Anti Phospholipid antibodies 50%
o RA Factor 40%
o Cryoglobulins
Biopsy for histopath and immunoflourescence
o Site most sensitive for biopsy (uninvolved,
unexposed site -back)
Radiological:- According to indication
o CXR, CT Scan
o Echocardiography
o ECG
o MRI
o CSF study
TREATMENT
Treatment is tailored according to the needs of Patient.
Goal of therapy:- Maintain optimal function e minimal therapy
General Measure:-
o Maintain morale, best rest, sun protection,
avoid smoking
o Avoid photosensitizers
o NSAID-for symptomatic relief
o Manage mental stress
o Sedatives
Topical Treatment:-
o Topical occlusives
o I/L steroids
o Tacrolimus
o Topical Salbutamol (0.5%)
o PDT laser
o CO2 laser
o UV Protection
Systemic Options:-
o First Line Options:-
LONG CASES 77

▪ Anti Malarial
▪ Systemic Steroids
o 2nd Line Options:-
▪ Methotrexate
▪ Dapsone
▪ MM
▪ Retinoids
o Further Options:-
▪ Colchicine
▪ Thalidomide
▪ IV Immunoglobulin
▪ Danazol
▪ Plasmapheresis
▪ UVA 1Therapy
Antimalarials:-
Indications:-
▪ When local therapy is ineffective
▪ Mutilating, disfiguring lesions
▪ Greater extent of disease
▪ No contraindications
o Take 3 weeks for showing full benefit
o After six month therapy, taper to lower dose
o HCQ 400mg/day
o Chloroquine 250mg/day
o Mepacrine 100mg-300
Corticosteroids:-
Indications:-
o For Acute flares
o 2-4 weeks then taper and maintain 5-
15mg/day dose
❖ Other Options:-
o Methotrexate – 10-25mg/week
o Acitretin – 25-50mg/day
o Colchicine – 0.5-1mg/day
o Dapsone – 200mg/day
LONG CASES 78

o MM-2g/day
o Thalidomide-400mg/day initially then 50-
100mg/day
o IV Immunoglobulin 2g/kg x 3days-monthly
o Azathioprine
o Cyclophosphamide
o Plasmapheresis
o ECP
o Biologics (Ustekinumab, Rituximab,
Belimumab 10mg/kg)
❖ Others:-
o Danazol 400mg (Premenstrual flares)
o UVA1 (340-400nm)-60KJ/m2 3time/week
o Low fat, high marine diet

Viva Questions

Q . Name antibodies e percentage?


▪ ANA 95%
▪ Anti Ds DNA 60%
▪ Anti Smith 25%
▪ Anti RO 30%
▪ Anti La 15%
▪ Anti Histone
▪ Anti Ribosomal P 25%
▪ Antiphospholipid antibodies 50%
▪ RA Factor 40%
Q. What is Lupus Band?
▪ Band like staining pattern of IgG, IgM, C3, Fibrin at DEJ
Q. Which site of biopsy has highest sensitivity?
▪ Uninvolved, unexposed site (back)
Q. First line treatment options for SLE?
▪ Antimalarials
LONG CASES 79

Q. Name biological agents used in tx of SLE?


o Rituximab
o Belimumab (Anti-BLYS)
o Ustekinumab
Q. Difference between overlap syndromes and MCTDs?
▪ Overlap syndrome is a broad term which includes
multiple syndrome including MCTDs.
Other include:-
o SLE / Sjogren overlap
o PM/SCL 70 overlap
o SLE/LP overlap
Q. Name 2 most common drugs involved in drug
induced LE?
o 15-20% Procainamide
o 5-8% Hydralazine
Q. DLE conversion to SLE (%ages).
o Localized DLE – 6.5%
o Disseminated DLE – 22%
o Chilblain lupus – 15%
o Lupus Profundus – 35%
Q. Risk factors for DLE conversion into SLE?
o Disseminated disease
o Female younger than 40
o HLA-B 8 linkage
Q. How to monitor SLE?
Quarterly visit
o Urinalysis – BP
o CBC – RFTs
o ESR, Anti DsDNA
o Complement levels
Q. Indications of renal biopsy?
o Proteinuria > 0.5g/day
o RBCs in urine
o Casts
o Creatinine clearance (< 80ml/min)
LONG CASES 80

Q. What is Rowell syndrome?


o DLE/LE like rash
o EM like lesions
o Anti RO Antibodies
o Speckled ANA pattern
Q. Which method is used in defection of ANA?
▪ indirect immunoflourescence (Hep 2 cells are taken as
substrate)
Q. How many cases of SLE are ANA –ive?
▪ 5% cases
Q. Name staining patterns of ANA?
o Homogenous – membranous
o Speckled – Centromeric
o Nucleolar – Ground glass
Q. 15 years survival rate?
▪ 75 %
Q. What are extractable Nuclear antibodies?
o Anti RO/LA
o Anti Smith
o Anti U1RNP
o Anti JO1
o Anti SCL 70
Q. How is drug induced lupus different from idiopathic
lupus?
Drug induced lupus is
o Uncommon in black race
o More D44 linkage
o M:F ratio 1:4
o Old age group
o Antihistone Antibodies
o Renal, CNS disease is less frequent
o Anti DsDNA antibodies are absent
o Normal complement levels
Q. Name salivary glands?
o Parotid, Sublingual, Submandibular
LONG CASES 81

Q. Drugs for LE-safer in pregnancy?


o Prednisolone, Azathioprine, Cyclosporine
o Aspirin, NSAIDS (1st 2 trimester)
o Tacrolimus, Antimalarials
Q. What is direct Coomb’s test?
o RBCs are separated from blood and incubated e
antihuman globulin (Coomb’s reagent). If
agglutination occurs, test is +ive.
Q. What is LE cell test?
o LE cells are polymorphs, which have ingested material
from degenerating white cells. (test is +ive if 20-30 %
cells on slide are LE cells)
Q. What are hematoxylin bodies?
o They are dense, homogenous, basophilic particles that
are easily stained e hematoxylin.
Q. What is interface dermatitis?
o Combination of basal cell vacuolization
o Apoptotic keratinocytes (Civatte/colloid bodies)
o Inflammatory Infiltrate
Examples: LE, Dermatomyositis, LP, GVHD,
Lichen Striatus, PLC, Infective causes.
Q. Types of lupus Nephritis?
o Minimal mesangial
o Mesangial proliferative
o Focal proliferative
o Diffuse proliferative
o Membranous
o Advanced sclerosing
Q. Disorders in which Tin tack sign is positive?
o DLE
o Pemphigus Foliaceus
LONG CASES 82

Long Case 2 B

Dermatomyositis

History:- Similar to the one mentioned already.


In clinical examination do not forget to ask the patient to
1. Drink water
2. Examine musculoskeletal system
3. Optic, occulomotor, abducens nerve
Investigations:-
A. ESR ↑
B. Serum electrolytes
C. Muscle Enzymes:-
o Aldolase- (converts sugar→energy) present in liver,
muscles.
o CPK-(ATP→creatine) present in heart brain and
muscles (most specific)
o LDH-(lactate→pyruvic acid) Present in all living cells
o AST- (Present in muscles, liver, heart, RBCs)
D. Autoantibodies:-
o ANA (60-80%)
o Myositis specific antibodies:-
o Anti JO 1 antibodies
o Anti SRP antibodies
o MI2 antibodies (most specific)
o With overlap features:-
o Anti RO/LA antibodies
o Ant U1RNP antibodies
o Other antibodies:-
o Anti PM1
o Anti KU
o PL 12
o Anti P155/140 (Carcinoma associated)
LONG CASES 83

o Anti CADM 140 (Amyopathic patient)


o Newer antibodies:-
o Anti NXP2 antibodies
o Anti SUMO antibodies
o Anti MDA-5 antibodies
o Anti TIF1 
E. Electromyography
o Spontaneous fibrillation
o Complex repetitive discharges
o Short duration motor potential
F. Skin Biopsy e Immunoflourescence
o Atrophic epidermis
o Interface dermatitis
o Basal layer degeneration
o Pigmentary incontinence
o Colloid bodies
G. Muscle Biopsy:-
o Perifascicular muscle atrophy
o Perivascular infiltrate
o Vacuolar degeneration
o Loss of transverse striations
H. Imaging and related Investigations
o Muscle MRI
o MR Spectroscopy (Amyopathic DM)
o Barium Swallow
o Chest, Abdomen, Pelvis CT Scan
o U/S Abdomen
o Chest x-ray
o Pulmonary function tests
I. To rule out underlying malignancy:-
o Stool for occult blood
o CP
o Biochemistry
o Pap smear
o Tumor markers
LONG CASES 84

o Transvaginal scan
o Mammography
TREATMENT
• General Measures:-
o Rest
o Exercise
o Indomethacin for pain
o Photo protection
o Topical Tacrolimus or steroids
o Prevent aspiration pneumonia
o Monitor electrolytes
o Calcium supplements
o Alendronate 70mg/week
• Systemic Therapy:-
o Start on Prednisolone 1mg/kg
o Start on avg 60mg/day then taper the dose and
maintain 5-15mg/day

o If no improvement CPK in 4-6 weeks

o Consider Pulse therapy for 3 months

o If no response after 03months consider 2nd option
o Antimalarials are indicated for skin rash
2nd Line Options:-
o Azathioprine
o Methotrexate 25mg/week
o Cyclophosphamide 100mg/day (lung disease)
Other Agents:-
o IV Immunoglobulin
o Rituximab
o TNF inhibitors
α
o Tocilizumab (Anti IL6 antibody)
o MM 2g/day
o Plasma exchange
LONG CASES 85

o ECP
Sirolimus-newer therapy
For Dysphagia – Diltiazem 30mg TDS
For Calcinosis- Al-Oxide 2.4g/day

Viva Questions

Q. Classify Dermatomyositis:-
a. Adult Onset
o Classic
o e malignancy
o Overlap syndrome
b. Juvenile
o Classical
o Amyopathic
c. Polymyositis
o Alone
o Overlap syndrome
o e malignancy
d. Inclusion body myositis.
Q. Name muscle enzymes involved in DM?
o Aldolase (convert sugar→ATPs)
o CPK (convert ATP→Creatine) *most specific
o LDH (convert lactate → pyruvic acid)
o AST (asprtate aminotransferase)
Q. Most specific antibody in DM?
o Anti Mi 2 antibody
Q. What EMG findings do u expect?
o Spontaneous fibrillations
o Complex repetitive discharge
o Short duration motor potential
Q. New antibodies in DM?
o Anti NXP-2
LONG CASES 86

o Anti SUMO
o Anti MDA5
o Anti TIF1 
Q. How do you categorize the disease as Amyopathic?
o Absence of muscle weakness > 6months
duration
o Normal muscle enzyme > 6months duration
Q. Poor prognostic features?
o Delay in initial treatment > 6months
o Malignancy
o Greater weakness at presentation
o Respiratory muscle weakness
o Interstitial long disease
o Cardiac involvement
o Dysphagia
Q. What is Anti-synthetase syndrome?
o One antisynthetase antibody
o Interstitial lung disease
o Polyarthritis
o Inflammatory myositis
Q. What are mechanic hands?
o Non inflammatory kyperkeratosis of hands/feet,
callosities
Q. Differences of Juvenile Dermatomyositis?
o No increased risk of cancer
o ↑ Calcinosis
o Lipodystrophy
o Hypertrichosis
o Anti JO1 less frequently involved
Q. Bohan and Peter criteria.
1. Symmetrical proximal muscle weakness
2. Muscle biopsy evident of myositis
3. ↑ muscle enzymes
4. Characteristic EMG pattern
5. Typical Rash
LONG CASES 87

For Polymyositis
o For definitive diagnosis
o All 4 are required
o Probable (03)
o Possible (Any 2)

For Dermatomyositis
o For definitive no 5 + any 3
o For probable no 5 + any 2
o For possible no 5 + any 1

Long Case 2C

Systemic Sclerosis

(History Questions are same as other CTDs with little


modifications)
On Examination:-
o Vitals-anemia, hypertension
On skin examination
In Hands:- look for
o pitted scars
o atrophy
o sclerodactyly
o telangiectasias
o Finger tapering
o Raynaud’s phenomena
o Elicit prayer sign
Arms:- Edema, myopathy
Face and Scalp:- Loss of hair, hidebound skin, salt and
pepper appearance, loss of forehead wrinkling, nose
breaking, puckering of mouth
Difficulty in pulling down lower eye lid
Legs:- Vasculitic lesions, patellar crepitus
LONG CASES 88

Musculoskeletal System:-
o Elicit crepitus
o Payer/Inverse prayer sign
o Raising hands above shoulders
o Standing from squatting position
o Ask the patient to drink water
(Focus on respiratory system and CVS in addition to
above)
Management:-
Investigations:-
o ESR ↑ed
o Blood CP = (anemia, thrombocytopenia)
o Hypergammaglobulinaemia
o Complement levels
o Renal and liver function tests
Antibody Profile
1. SCL 70 (Frosted glass appearance) 22%
2. Anti centromere antibody 30%
3. ANA-80%
4. Anti U1RNP antibodies (overlap syndrome)
5. Anti histone antibodies
6. Anti RNA polymerase antibodies
7. Anti PM-SCL antibody (Polymyositis and scleroderma
overlap
Histopathology with Immunoflourescence
o Dermoscopy (nail fold capillary exam)
o Infrared thermography For Raynaud’s
o For measuring skin thickness:- U/S, tomography,
durometry
Imaging and Related Investigations:
Respiratory System:-
o Chest x-ray, high resolution CT scan
o Pulmonary function tests
Musculoskeletal System:-
o Hand x-ray (pestle and mortar defect)
LONG CASES 89

Cardiovascular System ECG, Echo, 24hours Holter’s


monitoring
GIT Endoscopy, esophageal manometry, Ba-follow through
and enema
Treatment:-
General Measures:-
o Avoid smoking
o Exercise
o Keep hands warm, soak in water for 5mins daily
o Breathing exercises
o Job modification
o Duoderm dressing for finger ulcers
Treatment of Reynaud’s phenomena
o Job medication
o Topical nitroglycerine
o Tincture Benzoin Co (10%)
Treatment in Mild Cases:-
o Nifedipine 10-20mg 4times/day
o Prazosin – 1mg TDS
o Clonidine 0.5mg/day
o Losartan
o Diltiazem
o Vit C/E
In severe Cases:-
o Prostaglandin E, IV (0.5-2mg/kg)
o Oral Iloprost (100-300mg/day)
o Pentoxyphylline (0.4-0.8g/day)
o Sildenafil
o Bosentan
o Heparin, Clopidogril
o Botulinum toxin
o Sympathectomy
o Radial microarteriolysis
For Hypertension Captopril 25mg BD
For Esophageal Spasm
LONG CASES 90

o H2 receptor blockers
o Diltiazem 30mg TDS
For Facial Tightening (0.025-0.05% topical tretinoin)
Telangiectasias Co2 Laser or PDL
Fecal Incontinence Bulking agents, oral supplements
Diarrhea-Constipation- Laxatives, rotating antibiotics,
loperamide
Renal Crisis ACE inhibitors, Iloprost (avoid high dose
steroids)
Pruritis Ketotifen, montelukart, PUVA, steroids
Definitive-Systemic Therapy
Treatment (From Rook-9th Edition)
o IcSSc
Treatment of vascular complications

o dcSSc

Immunosuppressive and potential antifibrotic


therapies
o Overlap SSc

Manage-according to severity of dominant overlap


feature
o Severe /Active Skin Involvement:-
MMF, MTX, Cyclophosphamide, Rituximab, IV Ig
o Treatment of severe organ based-complications:-
Lung, heart, kidney-MMF/Cyclophosphamide,
Rituximab

Consider hematopoietic stem cell-transplantation


(In poor prognosis patients)

Viva Questions
LONG CASES 91

Q. Name antibodies in SS?


o SCL 70 (22%) → severe disease
o ANA (80%)
o Anticentromere antibody (30%) (associated e GORD,
CREST syndrome)
o Anti U1RNP antibody (overlap syndrome)
o Anti RNA polymerase 3 antibody (renal crisis,
malignancy, low mortality)
Q. Name scleroderma spectrum disorders?
• Morphea
• Eosinophilic fasciitis
• Scleromyxedema
• Nephrogenic systemic fibrosis
• Acrodermatitis chronica atrophicans
• Grafts vs host disease
• Lichen sclerosis
• PCT
• Myxedema
• Amyloidosis
Q. Name drugs causing scleroderma like changes?
• PVS
• Bleomycin
• L-tryptophan
• Silicone implants
• Pentazocin
• Urea formaldehyde
• Vit K
Q. Name bony deformity seen on x-ray (hands)?
o Pestle and mortar deformity
Q. What is MRSS-Its significance?
• Modified Rodnan skin score
• 17 different body sites are assessed for thickness and
are given score (0-3), maximum points is 51
• Significant score > 14 is associated e diffused disease
LONG CASES 92

Q. First line tx for diffuse skin disease?


o Methotrexate
Q. Treatment of choice for renal disease?
o Captopril (Capoten) 25mg BD
Q. Drugs used for severe Raynaud’s e doses?
→ Pentoxyphylline (0.4-0.8g/day)
→ Iloprost (100-300  g/day)
→ Bosentan (Bosmon 62.5mg BD)
Other agents include sildenafil, antithrombotics, and
topical glyceryl trinitrate
Q. New investigations for measuring skin thickness?
o U/S tomography, durometry
Q. Name Antifibrotic drug/agents?
o D-Pencillamine
o Minocycline
o PUVA
o UVA 1
Q. How to differentiate generalized morphea from SS?
o Raynaud’s is uncommon in morphea
o Trunk involvement is more in morphea
o Sparing of hands/fingers
o Nail fold capillaroscopy is normal
Q. How to do follow-up?
Annual Echo, Lung function tests, assessment of
systemic features.
Q. Clinical sign/symptoms of pulmonary hypertension?
o Dyspnea, Fatigue, Syncopy, Chest pain, Ankle edema,
Ascites, Chest pressure
Q. What kind of pigmentation do you see in SS?
o Salt and pepper pigmentation
Q. Bad Prognostic indicators?
o Male
o HLA-B8 disease
o Diffused disease
o Renal, Cardiac involvement, respiratory involvement
LONG CASES 93

o Decreased CMI
o (Five years survival-34-80%)
Q. First line Tx of Cardio respiratory disease?
o Induction phase → IV Cyclophosphamide + oral MM
e 10mg Prednisolone

Maintenance phase oral MM or Azathioprine

Resistant cases → Rituximab
Q. Cause of Nephrogenic systemic fibrosis?
o Gadolinium-base contract media
Q. Peculiar histopath finding in LSA?
o Band of collagen hyalinization (Papillary dermis)
‫‪LONG CASES‬‬ ‫‪94‬‬

‫‪Long Case 3‬‬

‫‪Erythroderma‬‬

‫)‪(History Questions‬‬
‫‪1‬۔ یہ بیماری کب اور کیسے شروع ہوئی۔‬
‫‪ 2‬۔ کیا اس سے پہلے آپ بلکل ٹھیک تھے یا جلد کی بیماری چل رہی تھی۔‬
‫‪ 3‬۔ سرخی آہستہ آہستہ پھیلی یا ایک دم پھیل گئی۔‬
‫‪ 4‬۔ بیماری سر سے پیروں کی جانب پھیلی یا پیروں سے سر کی جانب۔‬
‫‪5‬۔ سرخی کے ساتھ بخار یا کپکپی طاری ہوئی جسم پر۔‬
‫‪6‬۔ کوئی سوجن محسوس ہوئی ہاتھوں یا پیروں پر۔‬
‫‪7‬۔ اس مرض کے ساتھ سر یا باقی جسم کے بال جھڑنے کی شکایت تو نہیں۔‬
‫‪8‬۔ ناخن جھڑے ہوں یا موٹے ہو گئے ہوں۔‬
‫‪9‬۔ ہاتھوں‪ ،‬پیروں کی جلد موٹی ہو گئی ہو۔‬
‫)‪(PRP‬‬
‫‪Now rule out major cases‬‬
‫‪1. Drugs‬‬
‫‪10‬۔ مرض شروع ہونے سے پہلے کوئی دوا لی ہو۔ انگریزی‪ ،‬حکیمی یا‬
‫ہومیو دوا؟‬
‫‪11‬۔ سرخی چھوٹےچھوٹے دانوں کی شکل میں شروع ہوئی۔‬
‫‪12‬۔ کوئی دم کی ہوئی مٹی تو جسم پر نہیں لگائی۔‬
‫‪2. Psoriasis:-‬‬
‫‪13‬۔ چنبل کی شکایت تو نہیں تھی‬
‫‪14‬۔ جسم پر خشکی رہتی ہے یا چھلکابنتا ہو۔‬
‫‪15‬۔ کوئی زہنی تناو تو نہیں ہے۔‬
‫‪16‬۔ ایک دم سے کوئی دوائی بند کی ہو؟‬
‫‪17‬۔ بخار‪ ،‬ملیریا سکون کی کوئی دوالی ہو۔‬
‫‪18‬۔ شعاوں سے کوئی عالج کیا ہو۔‬

‫‪3.‬‬ ‫‪Dermatitis:-‬‬
‫‪19‬۔ بچپن سے جلد خشک تو نہیں رہتی تھی؟ یا سر میں خشکی رہتی ہو۔‬
‫‪20‬۔ ناک‪ ،‬کان یا چھاتی پر خشکی یا سرخی رہتی ہو۔‬
LONG CASES 95

‫۔ چھینکیں آتی ہوں یا سردی میں جلد معمول سے زیادہ خشک ہو۔‬21
‫خاندان میں دمے کی شکایت ہو‬
4. Lymphoma:-
‫ وزن کم ہو گیا ہو یا جسم میں کوئی گلئی محسوس ہوئی ہو۔ رات کو‬،‫۔ بخار رہتا ہو‬22
‫پسینہ آتا ہو۔‬
‫۔ جسم کے چھپے ہوئے حصوں پر کوئی نشان تو نہیں تھے۔‬23
To rule our other causes
o (Scabies) ‫۔ خاندان میں کسی کو خارش تو‬24
‫نہیں۔‬
o (Pemphigus Foliaceus) ‫۔ جسم پر کوئی چھالے بنتے‬25
‫تھے‬

o (Ask other questions according to the presentation of


patient)
Perform relevant clinical examination
Focus on
o Lymphadenopathy, Cardiac output
o Types of scale
o Body surface involved e nail and hair changes
o (rest of examination on the basis of suspected case)
Management:
• Hospital admission
• Stop all the drugs
• Avoid taking biopsy till the acute flare is settled
• Monitor vitals
Investigations:-
o Blood CP-anemia, sezary cell count, eosinophils,
platelets count, ESR
o Serum lgE levels
o LFTs, RFTs, e electrolytes and BUN
o Serum albumin levels
o ECG
o CXR PA View
o Glucose level
o Urine R/E
LONG CASES 96

Disease specific investigations:- (According to the features)


o Skin scrapings – KOH exam (Scabies)
o HIV testing
o Patch / photopatch testing (contact dermatitis)
o Serum ANA, anti DsDNA (CTDs)
o Bone marrow exam (only if any abnormality in
peripheral blood film)
o U/S Abdomen
o Immunophenotyping
o TCR gene analysis
o CD4:CD8 ration
o RAST-Atopic dermatitis
o Others:-
o Occult blood in urine and stool, CT
Abdomen, Pelvis, Chest)
o Cervical smears / mammography (only if
indication)
Once Acute Phase is settled:-
o Take biopsy if clinically the diagnosis is not reached.
o Biopsy sample for
o Histopath (look for specific cause)
o Immunoflourescence
o Gene rearrangement studies
Histopathology is not conclusive in 50% of the cases
Treatment:-
o Hospital Admission
A. Nursing care and general measures.
o Temperature regulation (30-32 ͦ C)
o Cleaning and removal of Crust
o Daily bathing in Luke warm water (35-38
ͦ C)
o Insulin if glucose ↑ed
o Hourly record of pulse, temp, blood
pressure
LONG CASES 97

o Daily input and output chart


maintenance (Ideal output
0.5ml/kg/hour)
B. Monitoring Hemodynamic Changes.
o Maintain urine out (↑50-100ml/hour)
o Osmolality should be maintained under
1020
o Pulse under 120/min
C. Fluid / Electrolytes and Nutrition:-
o Protein diet 2-3/kg/day
o If gastric emptying is delayed (Gastric
Aspiration)
In Initial 24hours
o Human albumin (Human 20%) 1ml/kg
body weight
o Normal saline 0.75ml/kg
From next day volume of fluids should
be determined on the basis of previous
day’s output.
For Hypokalemia:- Inj KCL (40mmol/l) in
5% dextrose or saline.
D. Prevention of Sepsis:-
o Prophylactic Antibiotics
Indications:-
o Confusion
o Hypothermia
o ↑ bacterial count
(Topical Therapy ↓+ General measures)
o Bland emollients
o Mild topical steroids
o H2 receptor blockers
o Diazepam
o Artificial Tears
o Sedative Antihistamines-hydoxyzine
30mg 6 hourly
LONG CASES 98

Systemic Therapy:-
o Treat according to the cause!

Viva Questions

Q. Name rare causes of Erythroderma?


o Sarcoidosis
o Hailey-Hailey disease
o Bullous Pemphigoid
o Toxic Shock Syndrome
o LE
o Dermatomyositis
o GVHD
o Lichen Planus
o HIV
Q. Name clinical subtypes of Erythroderma?
o Wilson Brocq (dry e large scales)
o Hebra (day e small scales)
o Vesiculoedematous
Q. What is ‘Nose Sign’
• Nose + Para nasal areas are usually spared in
erythroderma
Q. Name few causes of Erythroderma in children?
o Atopic dermatitis
o Psoriasis
o Omenn syndrome
o Netherton syndrome
o Icthyosiform erythroderma
Q. Background causes in idiopathic erythroderma?
o Adult onset atopic dermatitis
o Prelymphomatous eruptions
o Drugs- overlooked by the patient
Q. What are causes of peripheral edema?
LONG CASES 99

o Capillary leakage ↑ vascular permeability factors


↑VEGF
o Hypoalbuminaemia
o Cardiac failure
o ↑ BMR
o ↓ Hepatic protein synthesis
Q. Amount of Transpidermal fluid loss?
o 400ml/day normal and it becomes 3L/day in 50% skin
involvement
Q. Causes of Glycosuria?
o Pancreatitis →↓ insulin secretion
o Stress and infection
o Insulin resistance
Q. Mortality rate in Erythroderma?
o 18-64% mortality
Q. Ideal hemodynamic parameter?
o Urine output ↑ 50-100ml/hour
o Osmolality 1020 (should be under 1020)
o Pulse < 120/min
Q. Name fluids e doses?
o Human Albumin 20% 1 ml/kg
o Normal saline 0.75ml/kg
o Inj KCl (40 mmol) according to electrolyte level
Q. Bad prognostic signs?
o Old age
o > area
o Thrombocytopenia
o ↑ BUN levels
o Drug e prolonged half life
Q. What happens to Afterload / Preload in Cardiac
failure?
o Preload increases in Cardiac failure
o After load ↓es in Cardiac failure
Q. Causes of Protein loss in erythroderma?
o ↑ Catabolism
LONG CASES 100

o ↓ Protein synthesis
o Dilution
o Shedding of scale
o Enteropathy
o Old age
Q. Amount of scale shed daily in Erythroderma?
o 9gms/m2
Q. Indications of systemic steroids in Erythroderma?
o Metabolic complications
Q. Normal Iron and Vit D levels?
o Vit D → 20-50ng/ml
o Iron → 60-170  g/dl (male), 30-126  g/dl (Female)
Q. What is dermatogenic enteropathy?
o Condition in which there is malabsorption,
steatorrhoea e skin rash.
Cause is gut edema
Q. What is Cardiac Output?
o Cardiac output = Heart Rate x Stroke Volume 72 x
70ml = 4900ml/min
Q. Name dermatophyte that can cause Erythroderma?
o Epidermophyton Floccosum
Q. What is dermatopathic lymphadenopathy?
Benign histiocytic infiltration in paracortical areas of
lymph nodes.
(Associated e erythroderma)
Q. What is Immunophenotyping?
o Labeling of WBCs with antibodies directed against
specific surface proteins
o Sample required (Blood, tissue, spinal fluid)
Q. Cause of fever in Erythroderma?
• IL1 released by damaged Keratinocytes
Q. Causes of Idiopathic erythroderma (Redman
Syndrome)?
o Adult onset atopic eczema
LONG CASES 101

o Prelymphomatous drug eruptions


o Drugs over looked by patient
Q. Causes of Erythroderma in %age?
o Eczemas – 40%
o Psoriasis – 25%
o Lymphomas – 15%
o Drugs – 10%
o Unknown – 8%
o Infections - 2%
Q. Bad Prognostic Signs?
o Old age
o More BSA
o Neutropenia
o Thrombocytopenia
o ↑ BUN
o Drugs e greater half life

Long Case 4

Mycosis Fungoides
LONG CASES 102

‫ کب سے ہے اور کیسے شروع ہوا۔‬،‫۔ کسی مسلہ کے ساتھ آنا ہوا‬1


‫ ۔ ان نشانوں کے بننے سے پہلے کوئی مسلہ درپیش تھا؟‬2
‫ ناک اور کان میں خارش یا چھاتی پر سرخی تو نہیں‬،‫۔ بچپن سے سر میں خشکی‬3
‫تھی۔‬
‫ پودے یا سورج سے الرجی تو نہیں۔‬، ‫ ۔ کسی درخت‬4
‫۔ نشان ایک جیسے رہتے ہیں یا تبدیلی رہتی ہے۔‬5
‫ جلن یا درد محسوس ہوتا ہے۔‬،‫۔ ان نشانوں میں خارش‬6
‫۔ کبھی سرخی پورے جسم پر آئ۔‬7
‫ ٹانگوں پر خشکی یا چھلکا بنا ہو۔‬،‫ رنگت کی تبدیلی‬،‫۔ کبھی زخموں پر چھالے بنے‬8
‫۔ ٹانگوں پر خون کے رنگ کے دھبے محسوس ہوےہوں۔‬9
‫۔ کبھی جسم پر گلٹیاں یا پھوڑے بنے ہوں۔‬10
‫ رات کو پسینہ آتا ہو۔‬،‫ وزن میں کمی‬،‫۔ بخار رہتا ہو‬11
Proceed to other parts of history and focus on
o Quality of life
o Hospital stay
o Treatment options taken
o Personal history with systemic review
Management
Investigations:-
o Peripheral blood sample for following
o Hematology
o Biochemistry
o LDH levels
o CD4:CD4 ration (fluocytometry)
o Sezary Cell Count
o HTLV1 Serology
o TCR gene rearrangement studies (blood/skin
both samples can be used)
o Imaging:-
o CXR PA View (If indicated by clinical picture)
o Staging CT scan (Chest, abdomen, pelvis)
o PET Scan
o Biopsies:-
o Skin biopsy for H-pathology and
immunohistopathology
LONG CASES 103

o Lymph node biopsy if central node > 1cm in


size and peripheral lymph node is >1.5cm in
size
Treatment Options
A. Early Disease:- First line options
IA-IIA
o Wait and watch
o Topical steroids
o PUVA therapy
o Topical chemotherapy – Meclorethamine and
carmustine
o NB-UVB patch stage
o Radiotherapy
2nd Line Options
o Oral retinoids
o INFα
o Low dose MTX
o PUVA + Retinoid
o PUVA + INFα
o Total skin electron beam therapy
B (LATE DISEASE)
o Tumor Stage IIB
First Line
o Radiotherapy
o PUVA + Retinoids
o PUVA + INFα
o TSEB
2nd Line
o Baxarotene
o Toxic therapy
o Single agent chemotherapy (Doxorubicin)
o Gemcitabine
(Stage III/IV and SS)
o First Line:-
o PUVA-thrice a week
LONG CASES 104

o ECP-2days/month
o Baxarotene ‘Targetrin’ 300mg/day
o @-INF 3MU 3/weeks-for may weeks
o 2nd Line:-
o TSEB
o Antibody therapy (Monalizumab)
o Single agent chemotherapy
(Vorinostat)
o Third Line:-
o Stem cell transplantation

Viva Questions

Q. Pathophysiology in nutshell?
Cumulative mutations P53, P14, P15, P16 mutations.

o Defective DNA repair
o Apoptotic pathways are defective

Genomic Instability → Failure of activation induced
cell death

Increased survival of T-cells
Q. Tumor phenotype of MF?
• CD3+, CD4+. CDRO45+, CD7-
Q. What is TCR-gene analysis, which technique is used?
(PCR-technique used, sample required is skin biopsy)
TCR-gene is located on chromosome 7. Encodes TCR
molecule which is present on surface of T-cells
responsible for detection of antigens. A TCR molecule
is made of α,β chain and γ and Delta chains.
Technique is used to detect clonal proliferation of T-
cells.
LONG CASES 105

Q. What is T-cell homing?


Process by which cells return to their organ of origin.
e.g from blood circulation → area of insult → back to
blood
Q. What is a Pseudo lymphoma?
Benign disorders with histopath mimicking
lymphomas.
(Cytological atypia is minimal)
Causes
• Septran. Beta blockers, ACE inhibitors,
antidepressants, Anti Rheumatic drugs,
Anticonvulsants.
• Tattoos, trauma ,Borellia infections,
vaccination acupuncture, insect bites
Q. Name biological used in tx of MF?
Zanolimumab (Anti CD4 therapy)
Q. What is dermatopathic lymphadenopathy?
Paracortical expansion with histiocytes (begin pattern)
Q. What is CHOP regimen? How is it given ?
o Cyclophosphamide 750mg/m2 given over 5days
and repeated every 3 weeks

o Hydroxydaunorubicin 50mg/m2
o Oncovin 2mg/m2
o Prednisolone 40mg oral
Q. Which method is used to measure CD4:CD8 ratio?
Fluocytometry
Q. Outline investigations?
Peripheral blood samples for
o Hematology – HTLV1 serology
o Biochemistry
o LDH levels
o CD4:CD8 ration
o Sezary cell count
o Skin for TCR gene analysis
LONG CASES 106

o Lymph node sample (if involved)


o CXR
o Staging CT chest, abd, pelvis,
o PET scan
o Bone marrow examination (if peripheral film is
abnormal)
Q. Differentials of Pock like scars?
o Lipoid proteinosis
o EEP
o Chicken pox
o Hydroa vacciniform
o Atrophoderma vermiculatum
o Lymphomatoid papulosis
o Papulonecrotic tuberculid
LONG CASES 107

Long Case 5

Chronic Actinic Dermatitis

History Questions:
History questions will be similar to the questions asked in a
patient of chronic eczema or cutaneous T-cell lymphoma-focus
on questions like-
‫ کتنی مدت میں باقی جسم پر پھیال۔‬،‫۔ شروع کب ہوا۔ پھیال کیسے‬1
‫۔ بچپن سے جلد کی الرجی کا یا خارش کا مسلہء تھا ؟‬2
‫ ناک اور کان کے اردگرد یا چھاتی پر خشکی رہتی ہو۔‬،‫۔ سر میں خشکی‬3
‫۔ کبھی سرخی پورے جسم پر آ ءی ہو۔ سورج میں جانے سے کیا فرق پڑتا ہے۔‬4
‫۔ نشانوں پر جلن یا خارش آتی ہو۔ جسم کی چپھی ہوءی جگہوں پر نشان بنتے ہوں۔‬5
‫۔ اس کے لیے کیا عالج کیا ہے۔ کونسی دوا کھای۔ اور سورج کی شعاعوں سے‬6
‫عالج کروایا کبھی۔‬
‫۔ اس بیماری سے زندگی پر کیا اثرات مرتب ہوے ہیں۔‬7
‫۔ ہسپتال میں داخلہ ہوا کبھی۔ اگر ہاں تو کتنا عرصہ۔‬8
→ Proceed to systemic inquiry and rest of the history portion.
Management
A. Investigations:-
▪ Blood CP (rule out sezary cells)
▪ Photo testing:- Monochromatic testing to UVB and
visible light
▪ Patch/ Photo patch Testing
B. To rule out differential diagnoses:-
o Photo testing is normal in photo aggravated
Atopic or seborrhoeic dermatitis.
o Phototesting is normal in ABCD but patch /
photopatch testing may be +ive.
o In Mycosis Fungoides = Phototesting is
normal
C. IF MF is suspected:-
o Histopathology e immunotyping
o TCR gene studies
LONG CASES 108

o LFTs, RFTs
D. Other tests:-
o ANA-to rule out SLE
o Blood/ urine porphyrin levels to rule out
Porphyria
Histopathology
o Spongiosis, acanthosis, upper dermal parivascular
lymphohistiocytic infiltrate (Acute Phase)
o Epidermal hyperplasia, histiocytes, eosinophils,
plasma cells, atypical large hyper chromatic cells e
convoluted nuclei.
Therapeutic Investigations:-
o LFTs
o RFTs
o CXR
o Urine R/E
o TPMT levels
o DEXA Scan
Treatment:-
o Photo protection (sunscreens, hat, umbrella)
o Identify allergens and avoid them
o Vit D intake
o Monitor disease through regular photo
testing
Definitive Treatment:-
First Line Therapy:-
o Very potent topical steroids
2nd Line Therapy:-
o Tacrolimus topical + systemic steroids
o PUVA therapy under steroid cover
3rd Line Therapy Options:-
o Azathioprine
o HCQ
o Cyclosporine
o MM
LONG CASES 109

o Danazol
o Nitrogen Mustard
o Thalidomide
o INF-α
o Infliximab

Viva Questions

Q. Classify cutaneous Photosensitivity disorders?


A. Idiopathic (Immunological Basis)
o PLE
o Juvenile spring eruptions
o Actinic Prurigo
o CAD
o Solar Urticaria
o Hydroa vacciniform
B. Genodermatoses:-
o XP
o Bloom syndrome
o Cockayne syndrome
C. Photosensitizers:-
o Exogenous
o Naproxen
o Diuretics
o 5α-ALA
o Dapsone
o PABA
o Tetracyclines
o Sulfonamides
o Retinoids
o Endogenous (Porphyrias)
D. Photo aggravated Disorders:-
o Psoriasis
o Atopic dermatitis
LONG CASES 110

o EM
o Urticaria
o Seborrhoeic dermatitis
o Rosacea
o Lymphocytoma
o Keratosis Pilaris
o Acne Vulgaris
o LE
o Melasma
o Bullous SLE
Q. CAD is predominantly a photosensitivity to?
o UVB-light
Q. What is Actinic Reticuloid ?
o In chronic phase of disease, there is
lichenification and pseudolymphomatous
picture in histopath, that is known as Actinic
Reticuloid
Q. What is Wilkinson’s triangle?
Triangular area of sparing behind ears (In CAD)
Q. Clinical Variants of CAD?
o Allergic contact dermatitis like
o Actinic Reticuloid
o Nodular Prurigo like changes
o Erythroderma
Q. Enumerate Investigations?
o Photo testing
o Patch/Photo patch testing
o To rule out LE-ANA
o To rule out porphyrias. Blood/urine porphyrin
levels
o To rule out MF
o Histopath e immunotyping
o TCR gene analysis
o LFTs/RFTs (assessing health status)
Q. What is a choromphore?
LONG CASES 111

o Part of molecule, responsible for its color.


Q. What is monochromator light testing (photo testing)?
(Tungsten-halogen bulbs are used)
o One cm area of skin is exposed to different doses
of UVB, UVA and visible light
o Areas are examined after 24 hours, and minimal
erythema dose is noted.
Q. Effects of UVR on skin.
Acute Effects:- (localized)
o Inflammation
o Tanning
o Hyperplasia
o Immunological changes
(↓ APCs)
(↓antimicrobial peptides)
o Photooncycholysis
Acute Systemic Effects:-
• Vit D synthesis
• Mood elevation (β-endorphin production)
Late effects
• Pseudoporphyria
• Photoging
• Photo carcinogenesis
Q. UVR and Visible light spectrum?
• UVC 100-290nm
• UVB 290-320nm
• UVA 320-400nm
• Visible 400-700nm
Q. Etiology of Actinic Prurigo?
o Delayed hypersensitivity reaction to both UVB
and UVA
o HLA DR 104 linkage (in 80% cases)
‫‪LONG CASES‬‬ ‫‪112‬‬

‫‪Long Case 6‬‬

‫‪EPIDERMOLYSIS BULLOSA‬‬

‫‪History Questions:-‬‬

‫کس عمر میں چھالے نکلنا شروع ہوئے۔‬ ‫•‬


‫کس حصے سے نکلنا شروع ہوئے اور پھیلے جسم کے کون سے حصوں پر۔‬ ‫•‬
‫گچھے کی شکل میں تو نہیں بنتے۔‬ ‫‪o‬‬
‫پیداءش کے بعد جسم پر سرخی تو نہیں تھی۔‬ ‫‪o‬‬
‫ٹھوکر لگنے سے نئے چھالے بنتے ہیں؟‬ ‫‪o‬‬
‫چھا ال کتنی دیر میں پھٹ جاتا ہے۔خود بیٹھ جاتا ہے یا اآپ پٹھا دیتے ہیں؟‬ ‫•‬
‫زخم ٹھیک ہونے کے بعد سفید رنگ کے دانے‪ ،‬رنگت میں کوئی تبدیلی تو نہیں‬ ‫‪o‬‬
‫آ تی جلد پر‬
‫موسم کی تبدیلی سے کو فرق پڑتا‬ ‫‪o‬‬
‫بال جھڑے ہوں یا ناخن میں کوئی فرق پڑتا‬ ‫‪o‬‬
‫انکھوں میں جلن‪ ،‬زخم یا منہ میں کوئی زخم بنے ہوں‬ ‫‪o‬‬
‫آواز میں تبدیلی‪ ،‬کھانے پینے میں دُشواری‬ ‫‪o‬‬
‫چھوٹے پیشاب یا پاخانے میں کوئی تکلیف ہوتی ہو‬ ‫‪o‬‬
‫سانس لینے میں کوئی تکلیف‬ ‫‪o‬‬
‫وزن میں کمی ‪ ،‬بھوک کم لگنا‬ ‫‪o‬‬
‫پٹھوں‪ ،‬جوڑوں میں کوئی تکلیف‬ ‫‪o‬‬

‫)‪(Proceed to other components of History and Examination‬‬


LONG CASES 113

Diagnostic Investigations:-
1. Electron Microscopy:-
It shows the level of blistering and ultra-structural
features
o In Simplex- Intraepidermal
o In Junctional- Lamina Lucida
o In Dystrophic – Beneath Lamina Densa
2. Ultra structurally:-
o In EB-Simplex- Tonofilament clumping
o In JEB- Sparse Hemidesmosomes
o In DEB- Anchoring fibrils are reduced
3. Antigen Mapping:- This technique is used to look for
reduced expression of antigenic components (Indirect
Immunoflourescence). It identify proteins that are
strongly expressed.
4. Specific Antibody Probes:-
Antibodies are used to demonstrate the absence of
certain antigens
5. Molecular Diagnosis:-
o Cheek mucosa or venous blood are used as
samples
o Prenatal diagnosis is possible
6. Next generation sequencing:-
o Sequence of all known genes in one go
(22000 known genes)
o Then identifying mutations
Treatment of Patient:-
Assess the severity of disease:-
▪ Age, extent of disease (body surface
area), types of blisters
▪ Treatments used already
▪ Look for any malnutrition-advise
serum albumin levels
▪ Order Hb levels and body iron
levels
LONG CASES 114

▪ Counsel the parents regarding


course and prognosis
Manage Pain:-
▪ Advise to avoid any trauma to skin
▪ Modify dressing and shoes accordingly
▪ Puncture the blister to drain the bullae while leaving
the roof intact
▪ Start e mild agents like Paracetamol and NSAIDS
▪ In severe cases use Opioids, both topical and systemic
For Puritis:-
▪ Use sedative antihistamines like hydroxyzine
▪ Prefer liquid formulations
How to manage Wounds:-
▪ Monitor the size of erosions through serial
photographs
▪ Clean wound e normal saline or chlorhexidine
solution
▪ Puncture at multiple sites to allow drainage
▪ For wound debridement use non-physical methods
(Multiple dressing are available)
Type of Dressings:-
Foams:- Bi-layered
▪ Outer layer – silicone
▪ Inner layer – polyurethane
(Mepilix)
Hydrogels:- used in wounds with minimum exudates
(Duoderm)
Alginate Dressing:-
▪ Calcium or sodium alginate dressings
▪ Made of polysaccharides
▪ Used in exudative wound
Modified Absorbent Pads:- They are plastic films
▪ They prevent dressing from adhering to
wound
LONG CASES 115

▪ They have perforated surface that allows


passage of exudates (Mesorb)
Contact Layers:- Porous, Silicone bared material,
▪ They can be left into the place for 7 days.
▪ Secondary gauze dressing can be placed
(Mepitel)
APLIGRAFT:- Bi-layered Dressing.
Inner layer:- Human fibroblasts + Bovine type1
Collagen
Outer Layer:- Human Keratinocytes
Treatment of Infection:-
o Skin swab for C/S.
o Rotate topical antibiotics to avoid resistance
o Oral antibiotics are given if there are signs of
systemic infection
(Look for any sign of malignancy)
MULTIDISCIPLINARY APPROACH
Dental Care:- (Refer to Pediatric Dentist)
o Treat dental caries
o Tooth implantation
o Use of mouth Washes
Gastrointestinal Complaints:-
o Calcium channel blockers for esophageal spasm
o Balloon dilatation for strictures
o Treat GERD
o Supplements
o Laxatives for constipations
Eyes:-
o Artificial Tears
o Topical antibiotics
Urological Complaints:-
o Refer to urologist for opinion regarding
urological complaints
Musculoskeletal System:-
o Exercise and physiotherapy
LONG CASES 116

o Treat osteoporosis
o Nutritional supplements
o DEXA Scan-to assess bone mineral density
o Calcium and Bisphosphonates
Innovative Therapies:-
o Gene therapy
o Genomic Editing
o Recombinant protein therapy
o Cell therapy with intradermal allogenic fibroblasts
o Cell therapy with intradermal mesenchymal
stromal cells
o Bone marrow transplant
o Natural gene therapy
o Inducible pluripotent stem cells
o Non-sense read through drugs.

Viva Questions

Q. Name diagnostic tests?


▪ Antigen maping
▪ Antibody probes
▪ Electron Microscopy
▪ Molecular diagnosis
Q. What is kindler syndrome?
o Kindler syndrome is a type of EB e blisters at
different levels (mixed) variant.
Q. Name dressings that can be used in wound care of EB
patient?
o Foams
o Hydrogel dressing
o Alginate dressing
o Modified absorbent pads
o Contact layers
Q. Name Tx options in case of non-healing wounds?
LONG CASES 117

o Autologous split-thickness skin grafts


o Allogenic keratinocytes
o Skin bioequivalent (Apligraft)
Q. Serious complication that can occurs?
o SCC development
Q. Name new Tx modalities in EB?
o Gene therapy
o Genomic editing
o Recombinant protein therapy
o Cell therapy e intradermal allogenic fibrobalsts
o Cell therapy eintradermal mesenchymal stromal
cells
o Bone marrow transplants
o Natural gene therapy
o Inducible pluripotent stem cells
o Non-sense read through drugs
‫‪LONG CASES‬‬ ‫‪118‬‬

‫‪Long Case 7‬‬

‫‪PSORIASIS‬‬

‫‪ ۱‬۔ شکایت کب سے ہے ‪ ،‬کس عمر میں شروع ہوئی‬


‫‪۲‬۔ کس طرح پھیال اور جسم کے کون سے حصوں پر ہے۔‬
‫‪۳‬۔ بچپن سےکوئی الرجی تو نہیں ۔‬
‫‪۴‬۔ کیا چھلکا بنتا ہے۔ اگر ہاں تو کس رنگ کے ہیں‬
‫‪۵‬۔ خارش ہوئی ہے۔ خارش کرنے سے نئے دانے بنتے ہیں؟‬
‫‪۶‬۔ ان کے شروع ہونے سے پہلے گلہ خراب تو نہیں ہوا۔‬
‫‪۷‬۔ کوئی ذہنی تنائوتھا۔‬
‫‪۸‬۔ کوئی دوا تو نہیں کھائی تھی جیسے کہ ملیریا ‪ ،‬بلڈ پریشر‪ ،‬بخار یا ذہنی سکون کی‬
‫کوئی دوا۔ سگریٹ نوشی تو نہیں کرتے۔‬
‫‪۹‬۔ جسم پر کوئی پیپ والے دانے بنے ہوں‪ ،‬بغلوں میں یا ناف پر ۔‬
‫‪۱۰‬۔ ناخنوں میں کوئی تبدیلی محسوس کی ہو۔‬
‫‪ ۱۱‬۔ سورج سے زخموں پر کیا اثر پڑتا ہے۔ سردی گرمی میں کوئی تبدیلی محسوس‬
‫کرتے ہیں۔‬
‫‪۱۲‬۔ کس چیز سے بیماری بڑھ جاتی اور کس چیز سے بہتری آتی ہے۔‬
‫‪ ۱۳‬۔ پیٹ خراب رہنا‪ ،‬وزن میں کمی‪ ،‬جسم میں کوئی گلٹیاں محسوس ہوئی ہوں۔‬
‫‪۱۴‬۔ جوڑوں میں درد‪ ،‬جوڑوں میں سوجھن سرخی رہنا‪ ،‬صبح اٹھتے وقت جوڑوں‬
‫میں اکڑن رہنا‪ ،‬اکڑن کتنی دیر میں ٹھیک ہو جاتی۔‬
‫‪ ۱۵‬۔ کبھی اس بیماری کے لیے شعائیں لگائیں‪ ،‬کوئی دوا ایک دم سے بند کر دی ہو۔‬
‫‪ ۱۶‬۔ کیا عالج کیا ہے‪ ،‬دوا سے کیا اثر ہوتا‪ ،‬کتنی دفعہ ہسپتال میں داخل رہے ہیں۔‬
‫‪۱۷‬۔ خاندان میں کسی کو یہ مرض ہے۔‬
‫‪Proceed to systemic review, ask about quality of life and‬‬
‫‪proceed to clinical exam.‬‬
LONG CASES 119

Management:-
Investigations:-
o Blood CP = anemia
o ESR
o LFTs, RFTs to assess the baseline status
o Chest x-ray (TB)
o Glucose Levels
o X-ray long bones (If bones are involved)
o RA-factor
o Lipid profile
Assessment of Patient:-
o Salford Psoriasis Index:-
Takes into account
▪ Physical extent
▪ Psychological difficulty
▪ Previous therapies as mean of measure of
difficulty in treatment
PASI:-
o Head (Erythema + Induration + Scaling) x 0.1 x area
o Upper limbs (E + I + S) x 0.2 x Area
o Trunk (E + I +S) x 0.3 x Area
o Lower Limbs (E + I + S) x 0.4 x Area
Total score for each feature i.e. in Erythema, induration
and scaling is (0-4).
Area score
1-9% -1
10-29% -2
30-49% -3
50-69% -4
70-89% -5
90-100% -6
NAPSI:-
o Nails psoriasis severity index:-
o Each nail has 4 nail bed scores and 4 matrix scores
o Total score for all nails = 80
LONG CASES 120

Matrix Features
1. Pitting
2. Ridging/grooves
3. Red lunula
4. Nail plate crumbling
Nail Bed Features:-
1. Onycholysis
2. Subungual hyperkeratosis
3. Splinter hemorrhage
4. Oil drop sign
DLQI:- (Dermatology Life quality index)
Ten questions are asked.
Options
▪ very – 3points
▪ A lot – 2points
▪ Little – 1 point
▪ Not relevant = 0 point
Interpretation:-
▪ 6-10 moderate effect on life
▪ 11-20 very large effect
▪ 21-30 extremely disturbed
Treatment
▪ Attention to general health
▪ Avoid smoking
▪ Mild sedation
▪ Change of environment (humid environment)
Treatment ladder ( From Rook 9th edition)
A Mild Psoriasis without Psoriatic arthritis
First Line
▪ Coal tar. Dithranol ,Vit D analogue ,topical steroids
2nd line treatment
▪ NBUVB
▪ PUVA
▪ Excimer laser
B Moderate to severe Psoriasis (without Psoriatic arthritis)
LONG CASES 121

First line
▪ NBUVB/ PUVA
2nd line treatment
▪ Acitretin, Cyclosporine ,MTX, Fumaric acid esters
▪ Apremilast
3rd line treatment
▪ Infliximab, Adalimumab, Etanercept, Ustekinumab
C Psoriasis with Psoriatic Arthritis
▪ First line MTX , Apremilast
▪ Second line Biologics
▪ Third line Combination therapy
D Psoriatic Arthritis
▪ First line NSAIDS, Sulphasalazine, MTX,
Leflunamide
▪ 2nd Line Biologics
▪ 3rd Line Golimumab, Certolizumab,
Ustekinumab
E Pustular Psoriasis
▪ First line Acitretin, Cyclosporine, MTX
▪ 2nd Line TNF@Antagonists
▪ 3RD Line Anakinra
Viva Questions
Salford Psoriasis Index
Takes into account
o Physical extent
o Psychological impact
o Previous therapies as mean of measure of difficulty in
treatment
How to calculate PASI?
HEAD ( E+I+S) ×0.1×Area score ranges from 1-4
UPPER limbs ( E+I+S) ×0.2× Area
TRUNK ( E+I+S) ×0.3×Area
Lower limbs ( E+I+S) × 0.4 ×Area
Area percentage is calculated as
1-9% 1
LONG CASES 122

10-29% 2
30-49 % 3
50-69 % 4
70-89 % 5
90-100% 6
Q. What does DLQI mean?
Dermatology life quality index
Total ten questions are asked, options include
1. Very 3
2. A lot 2
3. Little 1
4. NAD 0
There is maximum score of 30
Interpretation
6-10 moderate effect on life
11-20 very large effect on life
21-30 extremely disturbed life
NAPSI score
8 POINTS for each nail
Matrix Nail Bed
▪ Pitting Onycholysis
▪ Ridging/grooves Subungual hyperkeratosis
▪ Red lunula Splinter hemorrhages
▪ Nail plate crumbling Oil drop sign
Q. What are Ingram and Goeckerman’s regime?
Goeckerman’s regimen
▪ Tar bath with exposure to Ultraviolet light B.
INGRAM’s regimen
▪ After the tar bath, sub erythemas UVB is given and
plaques are covered with Anthralin in increasing
concentration (0.05-4%), repeated after 22 hours
Mechanism of action of Vitamin D analogues
Vit D receptor mediated effects
▪ Regulate proliferation and differentiation of
keratinocytes
LONG CASES 123

▪ Shift TH1 profile to TH2 profile


CASPAR Criteria for Psoriatic Arthritis
▪ Current Psoriasis 02 points ( 3+
score is needed)
▪ Dactylitis 1point
▪ Family history 1 point
▪ RA factor negative 1point
▪ Nail dystrophy 1 point
▪ Juxtraarticular new bone formation 1 point
Drugs aggravating Psoriasis?
▪ Lithium, Beta blockers, ACE inhibitors, Antimalarials,
NSAIDS, Withdrawal of steroids and cyclosporine,
OCPs.
Associations of Psoriasis
▪ BP, Crohn’s disease, NMSC, lymphoproliferative
diseases, Gout,Hypocalcemia, Osteomyelitis
Uncommon Variants of Psoriasis?
▪ Verrucous
▪ Follicular
▪ Lichenoid
▪ Linear
▪ Zonal
▪ Sebopsoriasis
▪ Mucosal
▪ Ocular
Criteria for usage of Biologics
▪ Chronic psoriasis ≻6 months old
▪ Failure to respond /contraindications to 2 systemic
therapies
▪ DLQI ≻ 10
▪ PASI ≻ 10
Aggravating factors for Psoriasis
▪ Coal tar ,dithranol
▪ Infections
▪ Pregnancy
LONG CASES 124

▪ Hypocalcemia
▪ Hypothyroidism
▪ Withdrawal of steroids and cyclosporine
▪ Iodide, lithium, progesterone, terbinafine
▪ Salicylates
▪ Phenlybutazone
Association of Psoriatic Arthritis
▪ CARD 15 gene mutation
▪ HLA Cw6,B17, B13, DR3
▪ HLA B 27
▪ Hep C, HIV, Enterobacteria ,Streptococci
▪ Trauma
Most common pattern of Psoriatic Arthritis
▪ Predominantly peripheral mono or asymmetric
oligoarthritis ( 30%)
Psoriatic variants in children
▪ Guttate Psoriasis
▪ Toe cleft intertrigo
▪ Perleche
▪ Blepharitis
▪ Face involvement
▪ Pityriasis Amiantacea
▪ Follicular
▪ Pustular
▪ Napkin dermatitis
Radiological findings of Psoriatic Arthritis
▪ Enthesitis
▪ Periostitis
▪ Marginal erosions ( pencil in cup)
▪ Soft tissue swelling
How to differentiate between PsA and RA?
PsA RA
RA Factor 5 % ≻70%
Asymmetric symmetrical
DIP joints PIP and MCP joints
LONG CASES 125

Sacrum/lumbar area Cervical involvement


Juxtaarticular new bone formation Osteopenia
Vasculitis negative Vasculitis positive
Uveitis Sicca symptoms
Opera glass deformity Swan neck deformity
Z deformity
Boutonniere deformity
Poor prognostic factors in Psoriatic Arthritis
▪ Younger age
▪ Extensive skin involvement
▪ Polyarticular synovitis
▪ HIV
▪ Raised ESR
▪ Large effusions
▪ HLA B 27
Classify Biologics
▪ Monoclonal antibodies
▪ Interferon
▪ IV Immunoglobulin
What are novel therapies for Psoriasis?
▪ Phosphodiesterase 4 inhibitors
▪ JAK 2 Inhibitors ( under trial)
What is Apremilast?
▪ Apremilast is Phosphodiesterase 4 Inhibitor; it
increases cAMP levels within cells that antagonize the
effect of inflammatory cytokines.
How is Bath PUVA done?
▪ Affected areas are covered with psoralen solution
mixed with warm water and soaked for 15-20 mins
▪ Skin is gently dried
▪ Skin is then exposed to UVA
▪ Avoid sun exposure for 2-3 days after treatment
▪ Taken 2-3 times a week ( 10-60 cycles)
‫‪LONG CASES‬‬ ‫‪126‬‬

‫‪Long Case 8‬‬ ‫‪Leprosy‬‬


‫‪Viva Questions‬‬
‫‪ ۱‬۔ نشان کب سے ہیں۔ کتنے ہیں اور جسم کے کون سے حصے پرہیں۔‬
‫‪ ۲‬۔ شروعات کیسے ہوئی‪ ،‬دانا‪ ،‬دھبہ یا نشان تھے۔‬
‫سن تو نہیں ہے‪ ،‬سوئیاں تو نہیں چبھتی۔‬‫‪۳‬۔ زخموں پر محسوس ہوتا ہے۔ ُ‬
‫‪۴‬۔ خارش‪ ،‬جلن یا داغوں میں چبھن۔‬
‫‪ ۵‬۔ بخار تو نہیں ہوتا ان کے ساتھ‪ ،‬داغ وقت کے ساتھ کہاں کہاں پھیلے۔‬
‫سن ہوئے ہوں یا ان میں سوئیاں چبھتی ہوں۔‬ ‫‪۶‬۔ ہاتھ پاءوں ُ‬
‫‪۷‬۔روزمرہ کے کاموں میں کوئی دقت ہوتی ہو۔‬
‫‪۸‬۔ چلتے چلتے جوتا نکل جاتا ہو‬
‫‪۹‬۔ جسم پر پسینہ آتا ہے؟ بال تو نہیں جھڑے۔‬
LONG CASES 127

‫ چھالے بننا۔‬،‫ ۔ ہاتھوں میں انجانے سے زخم لگنا‬۱۰


‫۔ ناخنوں میں کوئی تبدیلی آئی ہو۔‬۱۱
‫ سرخی یا نظر میں کمزوری۔‬،‫ جلن‬،‫۔ آنکھوں میں درد‬۱۲
‫ آواز میں تبدیلی یا زکام رہنا۔‬،‫ خون آنا‬،‫ ۔ ناک بند رہنا‬۱۳
‫ منہ کے اندرزخم بنے ہوں۔‬،‫۔ ہونٹ موٹے ہوئے ہوں‬۱۴
‫۔ ان مسائل کے لیے کیا کیا عالج کیا۔‬۱۵
Type-I reaction
‫ کمزوری آ گئی ہو۔‬،‫ایک دم سے نشان زیادہ ہو گئے ہوں‬
‫ دوا کھانے کے بعد تکلیف بڑھ گئی ہو۔‬،‫یامنہ سوجھا ہو‬
Type-II reaction
‫ گلٹیاں بنی ہوں۔‬،‫چہرے یا جسم پر ا ُبھار‬
‫ جوڑوں میں درد یا گولوں میں درد یا سوجھن ہوئی ہو۔‬،‫ آنکھوں میں سرخی‬،‫بخار‬

Proceed to inquire other components, detailed treatment


history and ask about impact over quality of life
Management
Clinical Diagnosis ( 2/3 points are required)
• Anesthesia of skin lesions or over dorsal aspects of
hands and feet
• Thickened nerves
• Typical skin rash
ZN Staining
Smears can be taken from forehead, ear lobes, and extensors
of arms, buttocks and trunk (Detail given in section D)
Skin biopsy
Nerve biopsy
Lepromin test
• Strongly positive in TT
• Weak in Borderline Leprosy
• Negative in BB,BL,LL
Other tests
• Histamine tests
• Sweat test
• PCR
• Phenolic Glycolipids
LONG CASES 128

Treatment
Treatment Goals
o Chemotherapy (stop infection)
o Treat reactions(Minimize the risk of nerve damage)
o Educate the patient to cope with existing nerve
damage
o Social and psychological rehabilitation
o Treatment Protocol( Multidrug therapy)
Paucibacillary
▪ Dapsone 100mg daily
▪ Rifampicin ( Monthly 600mg supervised) 6 months
Multibacillary
▪ Rifampicin 600mg monthly ( supervised)
▪ Clofazimine 300mg monthly (supervised)
▪ Dapsone 100mg daily
▪ Clofazimine 50mg daily 12months
Ethionamide/Prothionamide
Used in case of Sulphone resistance 5mg/kg/day
Other Second line treatments
▪ Thiacetazone 2mg/kg/day
▪ Perfloxacin
▪ Septran
▪ Streptomycin
▪ Clarithromycin
Newer Regimen
Intermittent ROM Therapy
o Rifampicin 600mg
o Ofloxacin 400mg
o Minocycline 200mg
▪ Monthly for 12 months in MB Leprosy
▪ Monthly for 6 months in PB leprosy
PMM Therapy ( under trial)
▪ Rifapentine ,moxifloxacin, Minocycline
Special Consideration
Those who cannot take Clofazimine
LONG CASES 129

▪ Replace with 400mg Ofloxacin or 100mg Minocycline


Those who cannot take Dapsone
▪ If MB Leprosy Replace with Clofazimine for 6 month
▪ If PB leprosy Replace Omit Dapsone and continue
rest
If there is Rifampicin Resistance
▪ Replace with Ofloxacin 400mg or minocycline 100mg
If the patient is having Concomitant TB
For Paucibacillary patient
▪ Just add dapsone to maintenance regime
For Multibacillary patients
▪ Add dapsone in initiation phase
▪ Add Clofazimine and Dapsone in maintenance phase
Monitoring
▪ For PB Leprosy 2 YEARS
▪ For MB leprosy 5 YEARS
Treatment of Lepra reactions
o Reassurance
o Tranquillizers
o Analgesics
o Avoid bed rest
Steroids
o Start with 40mg and taper over 12 weeks
Manage neuritis
o Intraneural Lignocaine with hyaluronidase
▪ Surgical decompression
▪ Azathioprine and Cyclosporine can be other options
Type 2 Lepra reaction
Prednisolone 40mg, taper over 12 weeks
Clofazimine
o 300mg for 3 months
o 200mg for3 months
o 100 mg for 3 months
Thalidomide is no more recommended
Pentoxyphylline
LONG CASES 130

Other measures
▪ Bed rest
▪ Reassurance
▪ Antipyretics
▪ Treat dryness and other complications like orchitis
and eye damage

Viva questions

Tuberculoid vs lepromatous leprosy


Characteristics
Number 1-10 multiple
Distribution Asymmetric Symmetric
Edges well defined/hypopigmented Vague edges/slightly
hypopigmented
Autonomic loss early but limited late but extensive
Nerve enlargement Few nerves late but extensive
Systemic symptoms Absent In type 2 reaction
Acid fast bacilli Very few numerous

Skin lesions in Borderline leprosy


o Bizarre shaped, punched out lesions ,nodules, papules
o Lesions depend on the spectrum of borderline leprosy
Q . What is dissociative anesthesia?
o When one component of sensation is intact but other
is disturbed
o e.g. If there is loss of touch sensation but pain
sensations are intact
Q. Sites of body for taking smear? And why those sites?
o Forehead, ear lobes, chin, extensors of arms ,trunk
and buttocks
These areas are superficial “cold ‘areas so they have higher
yield.
Q. Difference in Fernandez and Mitsuda reactions?
Fernandez reaction
LONG CASES 131

▪ Seen at 48 hours
▪ Its hypersensitivity reaction to soluble component
(Protein that is released during preparation)
Mitsuda reaction
▪ Granulomatous response to particulate antigenic
material, seen at 4-6 weeks
Q. Name causes of nerve thickness/nerve damage other than
leprosy?
▪ Familial hypertrophic interstitial neuritis
▪ Progressive polyneuritis
▪ Amyloidosis
▪ HSAN
▪ Charcot Marie tooth disease
▪ Polyneuropathies (Diabetes,Alcoholism
,HIV,vasculitides, Arsenic poisoning)
▪ Congenital indifferentiation to pain.
Treatment Goals
1. Chemotherapy to stop infection
2. Treat Lepra reactions ( to minimize the risk of nerve
damage)
3. Educate the patient to cope with existing nerve
damage
4. Social and psychological rehabilitation
Q. How to prevent Methemoglobinemia?
▪ Add vitamin C
▪ Cimetidine can be added
Q. Side effects of Clofazimine?
Deposits in mesenteric lymph nodes creating a syndrome of
acute abdomen, mimicking pancreatitis.
Renal failure
Enteritis
Mahogany red-Charcoal black pigmentation of skin
It is anticholinergic, it can cause Icthyosis
Q. Name newer regimen with dosages?
ROM therapy
LONG CASES 132

▪ Monthly
▪ Rifampicin 600mg
▪ Ofloxacin 400mg
▪ Minocycline 200 mg
PMM regime
▪ Monthly
▪ Rifapentine 900mg
▪ Moxifloxacin
▪ Minocycline 200mg
Special consideration
When there is contraindication to Clofazimine
▪ Replace it with daily 400mg Ofloxacin or 100 mg
Minocycline
When Dapsone cannot be taken
▪ IF MB Leprosy-Replace with Clofazimine for 6 months
▪ IF PB Leprosy – Omit Dapsone and continue rest
If there is Rifampicin resistance
▪ Replace with Ofloxacin 400mg or Minocycline 100mg
Q. what is the role of immunotherapy in leprosy?
1. Immunotherapy enhances CMI
2. Eradicate persisters
3. Remove large pool of dead bacteria
Types of immunotherapy used
▪ Levamisole
▪ Zinc
▪ BCG
▪ Gamma interferon
▪ IL 2
Q. Type of vaccine available?
o BCG
o BCG with M Leprae (killed)
o M. Vaccae
Q . Precipitating factors of type 2 reaction?
• Injury
• Intercurrent illness
LONG CASES 133

• Stress
• Operations
• Pregnancy
• Mantoux test
• Anti leprosy drugs
• Parturition
• Iodides
Q. What are persisters?
• Bacteria that persist irrespective of proper treatment.
Q. How u define a case of leprosy?
• Person showing clinical signs of leprosy with or
without bacteriological confirmation
Q .Signs of relapse, how to differentiate a relapse from ENL?
o Erythematous papulonodular lesions
o Sites like abdomen
o Asymmetric
o ESR is normal
o No leukocytosis
o Persistent and non blanchable
IMPORTANT DRUGS 134

SECTION C

IMPORTANT DRUGS
IMPORTANT DRUGS 135

IMPORTANT DRUGS

1 Retinoids:-
A. Acitretin
Introduction 2nd generation retinoid, metabolite of
Etretinate.
(Advantage of shorter half life)
Classification of Retinoids:
o First Generation → Isotretinion and Tretinoin
o 2nd Generation → Acitretin and Etretinate
o 3rd Generation → Baxarotene, adapalene,
tazarotene
How does Acitretin work:

Regulates Keratinization

Decrease TH17cells ↓ ACITRETIN Decrease cell-cell bonding

(Act on family of nuclear receptors)


RAR and RXR types

(No immunosuppressive effects)


Major Indications:-
o Extensive psoriasis
o Palmoplantar pustulosis
o Darier disease
o PRP
o Congenital Icthyosis
o Erythrodermic Psoriasis
o Prophylaxis against NMSC
Dosage and Preparations (Brand Name Tigasor)
• Available as 10mg and 25mg
IMPORTANT DRUGS 136

• 0.5 mg/kg
• Maximum dose 75mg/day
• Response take 4 weeks
• With fatty meal for better bioavailability
Where they should be avoided.
o Pregnancy – avoid for 2 years after completion of
therapy.
o Double contraception during therapy
o Lactation
o Drug reaction with retinoids in past
o Hyperlipidaemia
Common Side Effects:-
Skin Specific
▪ Mucocutaneous dryness
▪ Exfoliative dermatitis
▪ Retinoid dermopathy
▪ Palmoplantar peeling
▪ Conjunctivitis
Others:-
▪ Teratogenic
▪ Hyperlipidaemia
▪ Suicidal tendency
▪ Sore eyes
▪ Hair loss
▪ Premature epiphysial closure
▪ Leukopenia
▪ Headache, drowsiness
▪ GIT upset

B . Isotretinoin:-

Mechanism of Action
IMPORTANT DRUGS 137

Anti-inflammatory

↓Sebum production Mechanism Antibacterial Against P.Acnes

↓Keratin Plugs
Important Indications:-
▪ Severe acne, nodulocystic acne
▪ Conglobate acne
▪ Acne excorie
▪ Late onset acne
▪ Rosacea
▪ Hydradenitis suppurativa
▪ Pyoderma faciale
Available Names:-
o Cap-Oratane 10mg
o Accutane 20mg
Q. How to calculate the commulative dose:-
o Isotretinion is given as 0.5-1m/kg/day
o Cumulative dose is established as weight(kgs) x 120-
150mg
= (dose to be given over 4-5 months)
Contra Indications:-
o Contra indications and side effects are mentioned
with Acitretin, pregnancy should be avoided for 3
months after completion of therapy.
(Retinoids Monitoring)
At first contact with patient advise
▪ Pregnancy test (females)
▪ Lipid profile
▪ Blood CPC (Baxarotene specific)
▪ Fasting Glucose
IMPORTANT DRUGS 138

▪ RFTS
▪ LFTS
Repeat monthly for 3-6 months then 3-monthly
* (Baxarotene comes with name Targetrin-Dose
300mg/day)

2. Dapsone:-
Introduction:- Chemically a sulphone 4', 4', diaminodiphenyl
sulphone
Q. How Dapsone works?

Competitive ← DAPSONE→
inhibition of enzyme ↓ ↓decrease
'Dihydropteroate' Inhibit the chemotaxis
synthetase release of
histamine
Blocks PABA's
incorporation into
Folic Acid

(Common Indications)

Autoummune bullous disorders:-


o Linear lgA disease
o CBDC
o DH
o Bullous Pemphigoid
o EBA
o Bullous SLE
IMPORTANT DRUGS 139

Neutrophilic Disorders:-
o Pyoderma Gangrenosum
o Sweet's syndrome
o Behcet disease
Others:-
o Leukocytoclastic Vasculitis
o Urticarial Vasculitis
o Granuloma Annulare
Q. How to take Dapsone? Brand Name (DAPSIN) 25mg
▪ 1-2mg/day
▪ Start as 50mg in first week
▪ Maximum dose 300mg/day
Where to avoid Dapsone?
▪ Sulphone hypersensitivity
▪ G6PD deficiency
▪ Anemia
▪ Acute porphyria
Major side effects:-
▪ Hemolytic anemia-but why??
▪ (RBCs generate NADPH in presence of G6PD- No
NADPH means hemolysis)
▪ Methemoglobinemia:- but why??
▪ Metabolites of dapsone act with oxyhaemoglobin and
converts Fe+2→Fe+3 and Fe+3 has ↓ capacity to carry
oxygen.
▪ Agranulocytosis:-
▪ Sulphone Syndrome- ↑LFTs, lymphadenopathy,
hepatitis, eosinophilia, morbilliform rash.
▪ SJS, TEN
▪ Mood disturbance
▪ Sensory and peripheral motor neuropathy
▪ Nephrotic syndrome (rare)
Monitoring
Initially
▪ FBC with retic count
IMPORTANT DRUGS 140

▪ RFTs
▪ LFTs
▪ Urinalysis
▪ G6PD Levels
CP with retic count, LFTs, RFTs
o Weekly for first month
o Monthly for 2 months
o Then 3 monthly
Q. How to prevent Methemoglobinemia and how to treat it?
o Prevention:- Cimetidine or Vitamin C
o Treatment:- IV administration of
methylene blue

3. Anti-Leishmanial Agents:-
Name the commercially available agents:-
o Na-Stibogluconate
o Glucantime
o Pentamidine
o Miltefosine
o Allopurinol
o Itraconazole
o Rifampicin
Antimony
They interfere with Bioenergetics of parasite leading to death
- Amount of elemental antimony
o Glucantime 5 ml vial – 85mg antimony / ml
o Pentostam – 100mg antimony / ml
Common side Effects:-
o GIT disturbance
o Metallic taster
o Abdominal and muscular pains
o Arrhythmias
o Liver and kidney damage
o Leukopenia
o Thrombocytopenia
IMPORTANT DRUGS 141

o Pancreatitis
(Monitoring)
o CP with platelets
o LFTs
o RFTs
o Amylase
o ECG
(Repeat weekly till tx)

4. Colchicine:-
Alkaloid extract
(Brand Name Colchicine)
Mechanism of Action ( Dose 0.5gm-1.5mg/day )
▪ Binds to tubulin and cause metaphase arrest
▪ Inhibit neutrophilic chemotaxis
Major Indications:-
▪ Gouty arthritis
▪ Behcet disease
▪ Aphthous ulcers
▪ DH
▪ Sweet's syndrome
▪ PG
▪ Linear IgA disease
▪ Systemic sclerosis
▪ Type 2-lepra reaction
Side Effects:-
o Nausea, vomiting, cramps
o Bloody diarrhea
o Myopathy
o Aplastic Anemia
o Agranulocytosis
o Hair loss

5. Systemic Steroids + Topical Steroids


IMPORTANT DRUGS 142

Topical Steroids:-
o Creams – emulsion of oil and water (oil in
water)
o Ointments- water in oil
o Lotions-hairy areas
o Paste – (powder + ointment)
o Gels – transparent, contain cellulose ethers
Contra Indications:-
▪ Skin infection, acne, perioral dermatitis, rosacea
Local Side Effects
▪ Worsening of infection, skin thinning, striae
formation, telangiectasias, contact dermatitis, perioral
dermatitis, acne, folliculitis, depigmentation,
hypertrichosis, tinea incognito
Systemic Side Effects
o Cushing syndrome
o adrenal suppression
o Rule of Hand pulp unit = 0.5gm (finger
tip unit)
(For one palm = 2 x pulp unit = 1gm)
o Hand = 1 FTU
o Arm = 3FTU
o Foot = 2 FTU
o Leg = 6 FTU
o Face/neck = 2.5 FTU
o Trunk (Front /back) 7 FTU

(European Classification)
MILD
o Hydrocortisone 1%
o Flucinolone acetonide 0.0025%
o Methyl Prednisolone
Moderate
o F. acetonide 0.006%
o Fluocinonide
IMPORTANT DRUGS 143

o Fluticasone 0.05%
Potent
o Beclomethasone
o Betamethasone velerate
o Triamcinolone
o Mometasone furoate
V. Potent
o Clobetasol propionate 0.05%
o Betamethasone Dipropionate 0.05%

Systemic Steroids
Mechanism of Action Analogues of endogenous steroid
hormones
Anti inflammatory:-
o Immunosuppressive
o Anti proliferative
o Vasoconstrictive
o Bind to intracellular glucocortoid receptors → then
complex binds to specific DNA sequences that
regulate gene transcription
o Inhibit prostaglandin and leukotriene synthesis by
blocking phospholipases
o ↓ Chemataxis
o Vasoconstrictive
o Stabilization of lysosomal membranes
Immunosuppressive effects:-
o Deplete langerhan cells
o Decrease immunoglobulin
o ↓IL 2
o Lymphocyte and eosinophil apoptosis

Classifications:-
Short Acting:
o Cortisone
o Hydrocortisone (8-12 hours)
IMPORTANT DRUGS 144

Intermediate Activity
o Prednisolone
o Methyl Prednisolone
o Triamcinolone (24-36 hours)
Long Acting
o Betamethasone
o Dexamethasone (36-54hours)

Conversion
- Cortisone – 25mg
- H. Cortisone – 20mg
- Methyl Prednisolone – 4mg
- Prednisone – 5mg
- Triamcinolone- 4mg
- Beta – 0.6-0.75mg
- Dexa – 0.75mg
Indications:-
o Atopic eczema, allergic/ irritant CD, erythroderma,
CAD, SD
o Drug rashes
o SLE, Dermatomyositis, mixed CT diseases
o Immunobullous disorders
o Vasculitides – PAN, wagener's granulomatosis , hyp-
vasculitis
o Neutrophilic dermatoses
o LP, SJS, hemangiomas, sarcoidosis, angiodema
Preparations:-
1. Prednisolone 5mg (Deltacortil)
2. Betamethasone 0.5mg (Betnesol)
3. Triamcinolone acetonide 40mg/ml ( K.Kort)
hexaacetonide 20mg/ml
4. M. Prednisolone 1g/2g (Depomedrol)
5. Hydrocortisone 100mg/ml (Solu cortef )
6. Dexamethasone 4mg/ml ( Decadron)
IMPORTANT DRUGS 145

Dose regimen with tapering


o Starting Dose – depends on severity
- For adrenal suppression → morning dose
(single)
- For androgen excess → night time low
therapy
(to suppress morning ACTH levels)
Adrenal Suppression
o Daily cortisol secretion is 20mg = 5mg Prednisolone.
o Short courses of high dose steroids don’t require
tapering – if less than 2 weeks of treatment.
o Once-dose reaches 7.5mg/day, reduction is slower 1-
2.5mg/week
During stress, infections, trauma, surgery
o Continue routine dose, then (extra dose- upto 3days)
o Who are taking 5mg or more > 4 weeks – risk of
suppression during stress
o Pt who have taken course less than 3 weeks – within
previous week – also require extra dose.
(Base-line Investigations)
o FBC
o RFTs with electrolytes
o LFTs
o Urinalysis for glucose
o BSR
o Fasting lipid profile
o BP / weight
o DEXA scan – lumbar / hips
o Growth chart for children
Monitoring
Following tests After one month then every – 2-3monthly
o BP
o Urine analysis (glucose)
o Urea / electrolytes
o DEXA after 6 month then 12 at month then yearly
IMPORTANT DRUGS 146

o Eye exam
Osteoporosis
o Inhibit osteoblast function
o Induce osteoclasts
DEXA is required to assess (BMD)
Scores: Normal (0 to -1)
Osteopenia (-1 to -2.5)
Osteoporosis (-2.5 or lower)
o How to prevent osteoporosis-
- No smoking / no alcohol
- Daily Ca, Vit D
- Alendronate 70mg weekly (prevent
osteoclastic activity)
- Milk ,yogurt
o For post-menopausal women → estrogen, hormone
replacement therapy
→ Testosterone in men
(Pregnancy Cat-C)
o Upto 40mg/day – lactation don't create problem
In early pregnancy → Cleft lip
In late pregnancy → IUGR

Contra Indications:-
o Absolute
o Latent TB
o Systemic Infection
o Relative
o Diabetes
o HTN
o Peptic ulcer
o Osteoporosis
o Liver disease
o Kidney failure
o Previous steroid myopathy
o Recurrent MI
IMPORTANT DRUGS 147

o Glaucoma
o Psychosis

(ADVERSE EFFECTS)
o Fluid / electrolyte imbalance :- - hypertension
- Na, water retention
hypokalemia
o Sudden Death: Pulsed therapy
(ventricular arrhythmias)
o Osteoporosis:- Cushing disease, diabetes
o Corticosteroid myopathy:- (proximal muscles
weakness, physiotherapy required)
o Peptic ulcer disease:- (give H2 receptor antagonists)
o Neuropsychiatric effects:- euphoria, irritability,
anxiety, sleep disturbance, cognitive impairment,
labile mood, depression, suicidal thoughts, mania.
o Cardiovascular:- (Thrombophlebitis)
o Ocular:- Glaucoma, cataracts
o Leukocytosis:- Decreased migration
o Infections:- Infections, zoster, TB, give zoster (VZIG) to
patients who received steroids in last 3 months
within (3 days of exposure) anaphylaxis, opportunistic
infections.
o Skin Side Effects:-Atrophy, striae, truncal acne,
telangiectasias, ↓collagen 1 synthesis
o Osteoporosis: More trabecular network (at risk)
Give Bisphosphonates, slow rate of bone turn over,
Ca – 1500mg/day
Vit D = 800IU
(Risk of Fracture)
1. Age
2. Sex
3. Low body mass index
4. previous fracture
5. parental history of hip fracture
IMPORTANT DRUGS 148

6. smoking
7. alcohol intake
o Avascular Necrosis:- (Osteonecrosis)
o Femoral head is most common effected
o Femoral condyles
MRI should be done
o Special consideration in Children:-
5mg/day – does not case – suppression
Tapering
For doses above 40mg/day

Decrease 10mg/week till dose reaches 40mg/day
(Stay on 40mg/day for 1week)

Decrease by 5mg/week till dose reaches 20mg/day
(Stay on 20mg/day for 1 week)

Decrease by 2.5mg/week till dose reach 5mg/day
(Stay on 5mg/day for 1 week)

Now taper by 1mg/week
Till they are off

6. AZATHIOPRINE
Mechanism of action:-
o Converts into 6-mercaptopurine

o Inhibits enzymes required for DNA synthesis

o Decrease proliferation of T and B cells


Dose – 1-3mg/kg/day
Commercial Name Imuran
Dose recommendations according to TPMT levels
Value Dose adjustment
• < 10 - Contraindicated
IMPORTANT DRUGS 149

• 10-25 - Dose 1-1.5mg


• > 26 - 2-3 mg

Indications:-
o Pemphigus Vulgaris
o SLE
o Dermatomyositis
o Bullous pemphigoid
o Urticarial Vasculitis
o Atopic dermatitis
o PLE
o Photosensitivity disorders
Contra Indications:-
o Hypersensitivity
o Severe infections
o Concomitant use of live vaccines
o Bone marrow depression
o Hepatitis
o Pregnancy and lactation
o Malignancy
Monitoring
o At Baseline:-
o FBC
o LFTs
o RFTs
o TPMT Levels
o Hep B and C Serology
o In females – (cervical screening)
Monitoring
FBC and LFTs monthly for 03months then bimonthly
Side Effects:-
o GIT disturbance
o Hypersensitivity reactions
o Hepatitis
o Bone marrow depression
IMPORTANT DRUGS 150

o Malignancy
o Infections
o Pancreatitis

7. CYCLOSPORINE
Introduction:-
o Calcineurin inhibitor
o Derived from a fungus Tolypocladium
Inflatum
Mechanism
o Calcineurin Inhibitor- Inhibits Calcineurin
dependent dephosphorylation

Activates specific nuclear factors

Prevents transcription of Pro-inflammatory cytokines.
o Anti lymphocytic
o Suppress langerhan cells
o Blocks neutrophilic chemotaxis
Commercial name 'Sandimmun Neoral'
Dosage:- * 1-5mg/kg/day
* start with 2.5mg/kg/day
* give in 2 divided dosages
Q. Where to use?
o Resistant Psoriasis
o Pustular psoriasis (safe in pregnancy)
o Atopic eczema
o Behcet's disease
o Pyoderma gangrenosum
o Lichen planus
Monitoring:-
At base line:-
o Cervical smear (in females)
o Measure BP
IMPORTANT DRUGS 151

o CBC
o RFTs with electrolytes
o Creatinine clearance
Then CBC + RFTs every 2 weeks for two months
then 3 monthly
- Creatinine clearance – 6monthly
- B.P – monitor in routine
o If diastolic BP> 95mmHg
- decrease dose by 1mg
- if still its elevated ,put the patient on
antihypertensive (ACE inhibitors)
- If creatinine:- >30% of normal value and
stays the same in 4 week-drug has to be
stopped
o Side Effects:
o Hypertension
o Nephrotoxicity
o Gingival hyperplasia
o Hirsutism and Hypertrichosis
o Infections
o Malignancies (CIN, PIN)
o Deranged LFTs
o Biochemical Derangements
o ↑ Hypertriglyceridaemia
o ↑ Hyperuricemia
o ↑ Hyperkalemia
o ↓ Glucose - ↓ ed
o ↓ Mg - ↓ed

8. METHOTREXATE
Introduction (Anti-Inflammatory-anti metabolite)
How does it work?
o MTX Inhibits the enzyme Dihydrofolate reductase
by blocking this enzyme it blocks the conversion
IMPORTANT DRUGS 152

of Dihydrofolate → Tetrahydrofolate (Blocks DNA


synthesis)
o Also inhibits lymphocytes.
Q. Where is this drug indicated?
o Plaque psoriasis resistant to conventional therapy
o Psoriatic arthritis
o Pustular psoriasis
o Psoriatic erythroderma
o Reiter syndrome
o Dermatomyositis
o Sarcoidosis
o Mycosis Fungoides
o Pemphigus Vulgaris
o Bullous Pemphigoid
o PRP
o Darier disease
o Systemic vasculitis
(Dose-Formulation)
o Start with test dose 2.5mg
o Route oral i/m, iv, sc
o If patient tolerates it, increase the dose weekly.
o Either once weekly dose
o Or in 3 divided dosages
o After 1 week monitor FBC
o If FBC is stable increase the dose by 2.5mg-5mg
weekly (maximum 30mg/week)
o Complete response by 6-12 weeks
o Add folic acid 5mg/day, except the dosage day.
Side Effects:
Dermatological Side Effects:
o Aphthous ulcers
o Alopecia
o Hyper pigmentation
o Toxic epidermal necrolysis
o Erythema recall after PUVA
IMPORTANT DRUGS 153

o Ulceration in psoriatic plaques


Others:
o Nausea, vomiting, abdominal pain
o Hepatotoxicity
o Nephrotoxicity
o Stress fractures
o ↓ sperm count
o Pancytopenia
o Teratogenic
In case of toxicity:-- Give folic acid 30mg I/V and repeat after
06hours
- Give charcoal immediately
- Give Na-Bicarbonate to alkaline urine
Monitoring:-
Get

• LFTs
• CBC
• CXR
• Hep B/C Serology
• PIIINP levels
• Urine analysis for glucose
• Pregnancy test in females
• HIV testing (in high risk-patient)
How to Monitor:-
- Repeat LFTs, RFTs, FBC weekly for 2weeks
after every dose increment.

(Once dose is stable)
Then 3 monthly
- PIII NP-every 3 months
- Liver biopsy after cumulative dose of 1.5g
Contraception:-
- (3 months after dose discontinuation).
IMPORTANT DRUGS 154

Q. Risk factors for liver fibrosis.


o Alcoholism
o Hepatitis
o Obesity
o Diabetes
o Impaired Kidney Function
o Cumulative dose of ≻2.5gms
Q. Significance of Pro-collagen 3 (P3NP)
o PIII N is derived from synthesis of type 3 collagen
o If 3 consecutive results of PIII NP > 4.2 ng/ml (high
risk of Cirrhosis)
Q. If LFTs are elevated, should we stop the drug?
o If LFTs > 3 times upper limit, stop the drug.

9. CLOFAZIMINE (Iminophenazine Dye)


Indications: - Leprosy, ENL, PG, Chronic DLE, Sweet's
Syndrome
Dose:- 100mg BD or TDS
Side Effects:- Icthyosis, brown pigmentation of skin,
pancreatitis like syndrome, photo toxicity

10. THALIDOMIDE (TNFɑ - inhibitor)


Indications:- ENL, Aphthi (resistant), Actinic Prurigo,
Kaposi Sarcoma, PG, Behcet disease, Bullous
Pemphigoid
Dose 100-300mg/day
Side effects:- Teratogenic, causes phacomelia, Peripheral
neuropathy, leukopenia, xerosis

11. PSORALENS
Naturally occurring photosensitivity agents
.Q How psoralens work?
o They ↓ immune cells
IMPORTANT DRUGS 155

o Cause melanocyte proliferation, transfer to


keratinocytes
o ↑ synthesis of tyrosine
Q. Types of Psoralens? Which one is better tolerated?
▪ 8-Methoxy Psoralens -
dose 0.4-0.6mg/kg
▪ 5-Methoxy Psoralens -
dose 1.2mg/kg → Better GIT
Tolerance
▪ Trimethoxy Psoralens -
dose 0.6mg/kg
▪ 1% leukodermin topical
Contra Indications:-
o Absolute: - XP, Gorlin syndrome, SLE,
Dermatomyositis, Bloom syndrome, previous
malignant melanoma
o Relative:- Hepatic and renal disease, pregnancy
,lactation, along with doxycycline
Side Effects:-
o Due to Psoralens:-
o Tanning
o Photo onycholysis
o Herpes reactivation
o Due to PUVA
o Photo aging
o PUVA lentigines
o Delayed tanning
o Melanoma
o Non-melanoma cancer
o Cataracts

12. ANTIMALARIALS:-
o How do they work?
▪ Stabilize lysosomal membrane (anti
inflammatory)
IMPORTANT DRUGS 156

▪ Decrease immune cells


(immunomodulation)
▪ Antioxidant
▪ Binds to porphyrin and facilitate its
urinary clearance –
(Photoprotective)
Dose Chloroquine 250mg/day, HCQ-200mg/day
Monitoring
o Baseline:-Vision, perimetry, slit lamp exam,
fundoscopy, serum transaminases
o Follow up:
- Repeat eye exam every six months.
- LFTs at 1st month, 3rd month and 6th month
Side Effects:-
o GIT Side Effects: Nausea, vomiting, diarrhea,
ALT ↑
o Mood Disturbance:- Altered mood,
psychosis,
o Eye:- Corneal deposits, retinal pigmentation,
field defects, maculopathy.
o Skin:- Lichenoid drug eruptions, purpura,
rash, blue black pretibial pigmentation ,
ochronosis, nail pigmentation

13. IVERMECTIN
Mechanism:- Blocks chloride channels of parasite
leading to paralysis.
Indications:- Scabies, pediculosis, Onchocerciasis,
Filariasis,Larva migrans
Dosage:- 200µg/kg or according to age
▪ 6 yrs – 12 yrs 1 Tab
▪ 12 yrs – 18 yrs 2 Tabs
▪ 18 yrs + 3 Tabs

14. ANTI TB THERAPY


IMPORTANT DRUGS 157

A. Rifampicin (Inhibit DNA dependant RNA polymerase of M-


Leprae)
Side Effects:-
o Hepatotoxic
o GIT disturbance
o Fatigue
o Red discoloration of urine and body
secretions
o Urticaria
o Skin rash, SJS, TEN
o Flue like syndrome
600mg/day

B. ISONIAZID:- (Inhibit mycolic acid synthesis of M-Leprae)


Side Effects:-
o Hepatotoxic 300mg/day
o Peripheral Neuropathy
o Acne form eruptions
o EM, SJS, TEN
o Lichenoid drug eruption
C. Ethambutal:-
Side Effects:-
o Optic Neuritis
15mg/kg/day
o Hypersensitivity
D. Pyrazinamide:-
o Gout
o Hepatotoxic
o 2g/day > 45kgs 1.5g/day < 45kgs
Commercially available preparations containing all 4
drugs are given as for
15kg - 1Tab
15-30 - 2 Tabs
30-45 - 3 Tabs
45+ - 4Tabs
IMPORTANT DRUGS 158

15. MYCOPHENOLATE MOFETIL


Mechanism:- (Imuxgen 500mg)
Inhibits enzyme inosine monophosphate
dehydrogenase leading to ↓ purine synthesis.
o Common Side Effects:-
1. GIT disturbance
2. leukopenia
3. anemia
* Avoid concomitant usage of MM and
Azathioprine (Both are purine blockers)

16. CYCLOPHOSPHAMIDE (Alkylating Agent)


Mechanism:- It irreversibly binds alkyl group to DNA
preventing its replication.

Dose 1-5mg /kg/day


Indications:-
o Immunobullous disorders
o Vasculitides
o CT disorders
o Pyoderma Gangrenosum
o Graft VS Host disease
Side Effects:-
o Cutaneous
▪ Nail Pigmentation
▪ Anagen Effluvium
▪ Teeth pigmentation
▪ Mouth ulcers
o Others
▪ GIT Disturbance
▪ TB Reactivation
▪ Gonadal Failure
▪ Hemorrhagic Cystitis
▪ Pancytopenia
IMPORTANT DRUGS 159

Monitoring:- CBC, CXR, Urinalysis, LFTs


o CBC Weekly for month, biweekly for 2-3 months
o Urinalysis Weekly for month, biweekly for 2-3
months
o LFTs Monthly for 3-6months then 3 monthly
o Urine Cytology – every 6 months

17. IMMUNO BIOLOGICALS:-


A. IV Immunoglobulin:-
o Introduction:-
Heterogenous human gammoglobulin, mainly
IgG with small amount of IgA and IgM, derived
from pool of human sera donated by thousands
of people.
Mechanism:-
o Suppress antibody formation
o Saturate Fc receptors on macrophages
o Neutralize super antigens
Commercial Name: - Sandoglobulins
Side Effects:-
o Headache
o Hemolysis
o Hypersensitivity reactions
o Fluid overload
o Renal failure
B. INTERFERONS
o Introduction:- Family of Glycoproteins
INFα – WBCs
INFβ – Fibroblasts
INF  (c) - Immune Cells
o Mechanism:-
- Antiviral
- Anti proliferative
- Immunoregulatory (↑ natural killer cells)
IMPORTANT DRUGS 160

o Indications:-
Commercial Name 'Leveron'
▪ Condylomata Accuminata
▪ Malignant melanoma
▪ Kaposi Sarcoma
▪ Warts
▪ Herpes Zoster
▪ T-Cell Lymphoma
▪ Keratoacanthoma

C. Monoclonal Antibodies
Classification
1. T-Cell Inhibitors
a. TNF@ - Antagonists
i. Infliximab
ii. Adalimumab
iii. Etanercept
b. Blocking T-Cell Activation
i. Alefacept
2. B Cell Directed Therapies
Rituximab (Anti CD 20)
3. Therapies directed Against Cytokines:-
• Ustekinumab (Anti IL12 / IL23)
• Anakinra, Rilonacept (Anti IL1)
4. Miscellaneous:- Omalizumab
- Ximab:- Chimeric monoclonal antibody
- Zumab:- Humanized monoclonal antibody
- Umimab:- Fully humanized monoclonal
antibody
- Cept:- Receptor Blocker
o TNFα Antagonists:-
o Infliximab (Remicade)
5mg/kg IV given over 0, 2, 6, 14 weeks
o Adalimumab (Humira)
40mg every 02 weeks for 12 weeks then weekly(SC)
IMPORTANT DRUGS 161

o Etanercept (Enbrel)
25mg S/C weekly x 12 weeks
Baseline Investigations:-
- FBC
- LFTs
- RFTs
- Urinalysis
- CXR
- Hep B and C
- βHCG levels
- Anti DsDNA antibodies
Monitoring:-
- Infliximab
FBC, LFT, RFTs, Urine R/E at 2, 6 week and
prior to infusion
- Adalimumab and Etanercept
FBC, LFTs, RFTs, Urine R/E at week 4, 12 and then 3
monthly
Contraindications
- Hypersensitivity
- Live vaccines
- Active sepsis
- CHF
- Poorly controlled diabetes
- Immunosuppression
Side-Effects
- Infections
- Cardiac failure
- TB
- CVS disease
- Demyelinating disorders
- Aggravation of Psoriasis
IMPORTANT DRUGS 162

- Allergic reactions
- Induction of Anti DsDNA
- SCC, lymphomas

Alefacept:-
- Blocks LFA 3 /CD2 interaction
- Dose-15mg I/M or S/C x 12 weeks
- Major side effect is decreased CD4 cells
UsteKinumab:-
- Prevent IL12 and IL23 from binding to their
receptors (Blocking T-Cell activation)
- Dose-90mg S/C or I/V
Rituximab:-
(Anti CD20) Ig4 I Antibody
Indications:-
o B-cell lymphoma
o RA
o Wagener’s Granulomatosis
o Bullous Pemphigoid
o Pemphigus Vulgaris
Dose 375mg/m2 weekly For 4-8weeks
Monitoring:-
At Base Line:-
- FBC, LFTs, RFTs, HIV Testing, Hep B/C
Serology, Pregnancy test, Immunoglobulin
levels.
- (Repeated prior to each infusion)

Newer IL1 inhibitors e indications:-


- Anakinra
- Canakinumab
- Rilonacept
Indications:-
IMPORTANT DRUGS 163

- Behcet Disease
- Muckle well syndrome
- Other antoinflammatory disorders

`18. APREMILAST
Introduction:- Inhibitor of enzyme phosphodiesterase 4.

Mechanism of Action:-
Phosphodiesterase 4 breaks down cAMP and is important in
inflammatory response, by inhibiting this enzyme inflammatory
response is blocked.
Indications:-
▪ Active Psoriatic arthritis
▪ Patients with extensive plaque psoriasis
DOSE
o Day 1 – 10mg in morning
o Day2 – 10mg in morning, 10mg in
evening
o Day 3 – 10mg in morning, 20mg in
evening
o Day 4 – 20mg in morning, 20mg in
evening
o Day 5 – 20mg in morning, 30mg in
evening
o Day 6 – and from there onwards
30mg BD
Common Side Effects:-
o GIT disturbance
o Upper respiratory tract infection
o Headache
o Weight loss
SPECIAL TEST & PROCEDURES 164

SECTION D

SPECIAL TESTS AND PROCEDURES


SPECIAL TEST & PROCEDURES 165

SPECIAL TESTS AND PROCEDURES

1 SKIN BIOPSIES
Shave Biopsy
Superficial skin biopsy done with size 10 or 15 blade
Indications
• Elevated lesions e.g. Compound Naevi
• No suture is required
Punch Biopsy
• Using 3-5mm punch biopsy is taken
• 1 or 2 sutures are required
Incisional Biopsy
• Elliptical portion containing fat is removed
• Indicated in conditions where is fat is involved or
lesion is too big to be excised
Excisional Biopsy
• Total excision of lesion, tumors can be completely
removed.
Curettage
• It is used in kyperkeratotic lesions like Seborrhoeic
keratosis or warts
Snip Biopsy
• Lesions are elevated and a snip is taken e.g.
Onchocerciasis.

2 DIASCOPY
A technique in which a glass slide is pressed against the lesion
to diagnose certain characteristics
• Urticaria Blanchable
• Vasculitis Non blanchable
• Lupus Vulgaris and TVC Apple jelly nodules
• Sarcoidosis Sand like grains
SPECIAL TEST & PROCEDURES 166

3 DERMOSCOPY
Technique
• Non invasive method of inspection of deeper layer of
skin
• Used mainly in differentiating Benign pigmented
lesions from Malignant melanoma

4 KOH EXAMINATION
• Used for diagnosis of fungal infections
• Clean the lesion with alcohol
• Scrape with size 15 blade
• Put the scrapings over a slide
• Add 1-2 drops of 10-20% KOH to dissolve keratin
• Heat a little and examine under microscope

5 TZANK SMEAR

Technique used to diagnose certain viral diseases, other


infections and genodermatoses
Fixative used Formol-Zenker Solution
Stains used Giemsa, H and E, Wright ,PAP stain

Procedure
• Blister is deroofed , base is scrapped with blunt end of
blade
• Smear is made over slide
• Apply Giemsa stain (Keep for 15 mins)
• Wash with water and examine
• Cytoplasm is stained blue and nuclei take red color

Important Indications
Pemphigus Vulgaris
SPECIAL TEST & PROCEDURES 167

• Tzank cells
• Large Acantholytic cells
• Large round keratinocytes with hypertrophic red
nuclei
• Edges are more densely stained (basophilic)

Herpes
• Multinucleate Giant cells
• Ballooning degeneration

Leishmaniasis
• LD bodies
• Light blue colored , ellipsoid bodies(20-30)
• Swarm of bee appearance within macrophages

Other indications
• Darier disease
• Hailey-Hailey disease
• SCC
• BCC
• Erythroplasia of Queyrat

6 ELISA
Technique used to measure the titer of antibodies in
autoimmune Bullous diseases and in diagnoses of various
infections.
Buffered solution is added to microtiter plates
Steps
▪ Primary antibody with conjugated enzyme is added
▪ Substrate for that enzyme is added and color of the
solution changes
▪ Higher the concentration of antibodies, stronger is
the color change
SPECIAL TEST & PROCEDURES 168

7 IMMUNOFLOURESCENCE

Direct Immunoflourescence
▪ Done on histopath sample
▪ Tissue is sectioned
▪ Tissue is then incubated with antibodies already
tagged with fluorescent molecules to allow
visualization under fluorescence microscope.
Salt split assay
▪ Punch biopsy is taken
▪ Its incubated with 5ml of 1mol/L NaCl at 40 degrees
Celsius for 24 hours
▪ Split forms at lamina lucida
▪ Epidermis is peeled off with fine forceps and rest of
the procedure is like DIF

Indirect Immunoflourescence
▪ Serum or urine is incubated with substrate like
monkey esophagus, rat bladder of human skin.
▪ Antibodies are added to substrate
▪ Anti human antibodies are added against that specific
antibody.

8 WESTERN BLOT (Immunoblotting)


▪ In first step target proteins are separated by
electrophoresis and transferred onto membranes
(Nitrocellulose membranes)
▪ Membrane is overlaid with primary antibody and then
secondary labeled antibody is added,
▪ Visualized through Fluorescence microscope.

9 IMMUNOADSORPTION

▪ Blood purifying technique


SPECIAL TEST & PROCEDURES 169

▪ Used to remove harmful antibodies with aid of SH-


membranes

10 EXTRACORPOREAL PHOTOPHERESIS
• First 8 MOP is given
• Peripheral blood is withdrawn
• RBCs and WBCs are separated by a machine
• RBCs are returned back while lymphocytes are
irradiated by UV light
• Irradiated blood is then returned to body
Mechanism
• T cell apoptosis
Indications
• T cell lymphoma, Connective tissue disorders,
Immunobullous disorders
Side Effects
• Hypotension, CCF, flushing, palpitations, infections

11 PHOTODYNAMIC THERAPY
Topical ( Alpha aminolevulinic acid)
Systemic ( hematoporphyrin X)

Above mentioned agents are given and then after six hours
lesions are exposed to red light
Indications
• BCC
• T-cell lymphoma
• Actinic Keratosis

12 PLASMAPHERESIS
Plasma filtration process, three methods are used
1. Plasma filtration
2. Continuous flow centrifugation
3. Discontinuous flow centrifugation
Process
SPECIAL TEST & PROCEDURES 170

• RBCs and plasma are separated


• RBCs are returned
• Plasma is reconstituted with replacement fluid and is
returned to patient
Complications
• Hypotension , transfusion reaction, decreased
immunity, hematoma formation

13 WOOD’S LAMP EXAMINATION


• Wood’s lamp uses a 360nm light source
• Wood’s filter is made of 9 % Nickel oxide and Barium
silicate
Indications with findings
o Tinea versicolor Dull yellow
o Dermatophytosis Yellow green
o Erythrasma Coral red
o Pseudomonas Pale blue green
o PCT Dark pink urine
o Vitiligo Chalky white

14 ZIEHL-NEELSEN STAINING
Staining
o Smear is made from specific sites
o Cover the slide with 1 % Carbol Fuchsin stain
o Heat the slide gently, avoid boiling
o Wash gently under running tap water and wash till
runoff water is clear
Decolorizing
o Cover with 1 % Acid Alcohol mixture for 10 seconds
o Rinse gently with water

Counter Staining
o Cover with counter stain 0.2% Methylene blue
o Rinse with water and let the slide dry in the drying
rack, now examine the slide.
SPECIAL TEST & PROCEDURES 171

You might also like